aamc 7r solutions

40
SOLUTIONS MCAT * Practice Test 7 MCAT MEDICAL COLLEGE ADMISSION TEST www.PrincetonReview.com

Upload: anne-ndoe-essono

Post on 08-Oct-2014

1.501 views

Category:

Documents


13 download

TRANSCRIPT

Page 1: AAMC 7R Solutions

SOLUTIONSMCAT*

Practice Test 7

MCATMEDICAL COLLEGE ADMISSION TEST

www.PrincetonReview.com

Page 2: AAMC 7R Solutions

MCAT Practice Test 7 Solutions2

MCAT Practice Test 7 SOLUTIONS

Edited, produced, typeset, and illustrated by Steve Leduc Director of MCAT Program Development The Princeton Review

CONTENTS PAGE

Physical Sciences solutions .....................................................3 Steve Leduc (Physics) Steve Leduc, Bethany Blackwell (General Chemistry)

Verbal Reasoning solutions ...................................................12 Jennifer Wooddell

Biological Sciences solutions ................................................32 Judene Wright (Biology) Bethany Blackwell (Organic Chemistry)

Copyright © 2004 by Princeton Review, Inc.All rights reserved.

*MCAT is a registered trademark of the Association of American Medical Colleges (AAMC).The Princeton Review is not affiliated with Princeton University or AAMC.

This document is for the exclusive use of Princeton Review course students,and is not legal for resale.

www.PrincetonReview.com

040721

Page 3: AAMC 7R Solutions

Physical Sciences Solutions 3

PHYSICAL SCIENCES SOLUTIONS

Passage I (Questions 1–7)

1. C. When looking for a correct Lewis structure, first check to see if a proposed structure at least accounts for the correct number of valence electrons. Each nitrogen atom has 5 valence electrons, and each hydrogen atom has 1. Therefore, the Lewis structure for N2H4 must account for (2 × 5) + (4 × 1) = 14 valence electrons. Since each line ( — ) in the structure represents a pair of electrons, the structure shown in choice A (with 6 lines and no lone pairs) accounts for 6 × 2 = 12 electrons, and the structure in choice B (with 6 lines and 2 lone pairs) accounts for (6 × 2) + (2 × 2) = 16, so neither of these can be correct. To decide between choices C and D, we now realize that a nitrogen atom needs just 3 electrons to complete its octet, so we expect each nitrogen to form 3 bonds as follows:

Therefore, the structure shown in choice C is best.

2. D. The Raschig process is given by Equation 1 in the passage. Because this equation is balanced, we can see that to produce 1 mole of hydrazine, 2 moles of ammonia are required. The mass of 2 moles of NH3 is 2[14.0 + 3(1.0)] = 34.0 grams.

3. D. According to the passage, the chemical formula of hydrazine hydrate is N2H4 · H2O. The mass of the hydrazine is 2(14.0) + 4(1.0) = 32.0 grams, and the mass of the water is 2(1.0) + 16.0 = 18.0 grams, so the total mass of the hydrate is 32.0 + 18.0 = 50.0 grams. Therefore, the percent by weight (or mass) of hydrazine in hydrazine hydrate is 32.0/50.0 × 100%.

4. A. The reaction given in this question shows the formation of hydrazine from its elements in their most stable form, so the enthalpy change ΔH° will be the standard enthalpy of formation, ΔHf°. According to Table 1, the value of ΔHf° for N2H4( ) is 50.6 kJ mol–1.

5. B. One way to compare the relative strengths of a pair of weak bases is to compare their Kb (basicity constant) values: the lower the Kb, the weaker the base. So, the fact that hydrazine is a weaker base than ammonia is reflected in the fact that hydrazine has a smaller basicity constant than ammonia; thus, choice B must be correct. Choices C and D can be eliminated since the number of protons a base can accept or its ability to hydrogen bond is unrelated to base strength.

6. C. The change in Gibbs free energy ΔG for the formation of hydrazine from its elements is the standard free energy of formation, ΔGf°. In order for a reaction to be spontaneous, the value of ΔG must be negative. However, according to Table 1, the value of ΔGf° is positive (149.2 kJ mol–1).

7. C. Gases are more disordered—and thus have a greater entropy—than liquids. In Equation 2, there are 0 moles of gaseous reactants and 7 moles of gaseous products. Therefore, the entropy increases (ΔS° > 0) because the number of moles of gaseous products is greater than the number of moles of gaseous reactants.

Passage II (Questions 8–13)

8. B. Choice A is incorrect, since the strong nuclear force binds protons and neutrons within a nucleus; it does not bind atoms to each other. Choice D is eliminated because Coulombʼs law simply describes the force between charged particles (in this case, between the positively charged ions and the negatively charged electrons); the law does not “prohibit” the motion of the ions. Choice C can be eliminated since the ions do feel an electric force when the electron sea is displaced. The passage states that when the electrons are displaced, electric fields act; but we know that whenever an electric field acts on a charged particle, the particle experiences an electric force (F = qE). Therefore, the answer is B. When we consider the force between an ion and an electron, each of these particles feels the same magnitude of force, F (by Newtonʼs Third law), but the effects will be very different here, because the masses of the particles are so different. An ion is much more massive than an electron, and as a result, the acceleration of the ion, aion = F/mion, is much smaller than the acceleration of the electron, aelectron = F/melectron.

Page 4: AAMC 7R Solutions

MCAT Practice Test 7 Solutions4

9. D. We know from our experience with the spring–block oscillator that the maximum elastic potential energy occurs when the spring is at its maximum stretch or at its maximum compression, which happens when the block is at either oscillation extreme. The same is true here. The maximum electrical potential energy occurs at the oscillation extremes, shown by Diagrams A and C in Figure 1. (Note that the potential energy would be zero in Diagram B.)

10. C. We use the equation given in the passage for f in terms of n: f ≈ 9.0n1/2. If n = 1018, then n1/2 = (1018)1/2 = 1018·1/2 = 109, so f ≈ 9.0 × 109 Hz.

11. A. Using the fundamental equation for waves, λf = v, we find that .

12. A. When the electron sea is at Point B in Figure 1, there is zero net force on the electrons (so choice B is wrong) and the potential energy is zero (so choice D is wrong). We can also eliminate choice C; not only does the passage make no mention of magnetic forces, but we also know that magnetic forces cannot alter the speeds of particles (magnetic forces do no work). As the electron sea passes through Point B and moves on to Point C, its speed decreases to zero. The answer must be A. Just as with the familiar spring–block system, the oscillator has its greatest speed—and thus its greatest momentum—as it passes through the equilibrium position. It is this momentum that causes the oscillator—in this case, the electron sea—to overshoot the equilibrium position (Point B in Figure 1) and move on to the other oscillation extreme.

13. B. At Position A, one of the oscillation extremes, the oscillator (the electron sea) has zero speed, and thus zero kinetic energy; all the energy of the system is in the form of potential energy. Then, as the oscillator travels through Position B, the equilibrium position, its potential energy is minimized and its kinetic energy is maximized. Therefore, in moving from Position A to Position B, potential energy is transformed into kinetic energy (choice A is backwards; B is correct). Choice C can be eliminated, since motion generates heat when thereʼs friction, but there is no friction here. (Furthermore, it is energy, not power, that may be transformed into heat.) Choice D can be eliminated since the electron sea is not at rest at Position B; on the contrary, it has its maximum kinetic energy (and thus maximum speed) and overshoots this position.

Passage III (Questions 14–19)

14. C. Choice A is eliminated since the first sentence of the passage makes it clear that silicon is not a rare element. The last sentence of the passage states that pure (elemental) silicon is nonreactive, so choice B is wrong. We can also eliminate choice D: silicon is a solid metal and thus exists in crystalline form. Choice C is best; the point of the second paragraph is that silicate minerals do not decompose easily.

15. A. First, eliminate choice D: a silicon atom contains 14 electrons, and choice D accounts for 15. The difference between choices A, B, and C is the description of the two electrons in the 3p subshell. The three lines after the “3p” in each of these choices denote the 3px, 3py, and 3pz orbitals. How do we place the two electrons in the 3p subshell into these three available orbitals? Hundʼs rule says that if more than one orbital is available, we place electrons with parallel spins into different orbitals before we pair them up (with opposite spins) in the same orbital. It follows that choice A is the correct configuration.

16. B. The silicon atom in SiCl3H has four bonds: one to each of the three chlorine atoms and one to the hydrogen atom. Only choice B describes a geometry with four bonds.

17. B. Elements in the same family have similar valence electron configurations and thus have similar chemical properties. Since potassium is a Group 1 metal, we eliminate choices C and D, which are both Group 2 metals. We also eliminate choice A, because even through hydrogen is in Group 1 in the periodic table, it is not considered a metal. Choice B, which like potassium is also a Group 1 metal, is best.

18. D. Choice A is wrong: There will be no ionic forces between these molecules because ions are created by metals reacting with nonmetals and all the elements in this molecule are nonmetals. Choice B is eliminated because covalent bonds are intramolecular, not intermolecular forces, and choice C is wrong since molecules of SiCl3H are incapable of hydrogen bonding.

19. C. The technique of distillation exploits the different boiling points of the components of a mixture to separate them.

Page 5: AAMC 7R Solutions

Physical Sciences Solutions 5

Passage IV (Questions 20–23)

20. C. If 3 neutrons are produced in the fission reaction, then the reaction is

In order to conserve mass number (the superscripts), it must be true that 236 = A1 + A2 + 3·1, so A1 + A2 = 233.

21. A. The reaction shown in choice A is the only one that conserves both mass number (the superscripts: A1 = A1 + 0) and electric charge [the subscripts: Z1 = (Z1 + 1) + (–1)].

22. B. Because Ra-226 has a shorter half-life (1600 years) than Pu-239 (24,000 years), Ra-226 decays faster, so choices C and D are eliminated. Since 24,000/1600 = 15, the half-life of Ra-226 is 15 times less than the half-life of Pu-239, so the rate of decay of Ra-226 is 15 times faster than the rate of decay of Pu-239.

23. B. Because density is mass divided by volume ( = m/V), it follows that volume is mass divided by density: V = m/. Dividing m = 106 kg by = 103 kg/m3, we get V = 103 m3. The volume of a cube is given by the formula V = s3, where s is the length of a side. If s3 = 103 m3, then s = 10 m.

Free-Standing Questions (24–27)

24. A. Multiply both sides of the first reaction by 2; note carefully that this does not change the value of E° for the reaction. Now, add the resulting reaction to the second reaction given in the question and then cancel the 4 e– that appear on opposite sides of the reaction arrows. The final result is the reaction for which we want . Adding the values of E° for the separate reactions, we find that = (+0.34 V) + (–1.23 V) = –0.89 V.

2 Cu2+ + 4 e– → 2 Cu(s) E° = +0.34 eV + 2 H2O → O2 + 4 H+ + 4 e– E° = –1.23 eV

2 Cu2+ + 2 H2O → 2 Cu(s) + O2 + 4 H+ = –0.89 eV

25. B. Since the temperature remains constant, we use Boyleʼs law, P1V1 = P2V2. With P1 = 1 atm, V1 = 10 L, and P2 = 500 atm, we get V2 = P1V1/P2 = 0.02 L. However, this assumes that the gas continues to behave as an ideal gas. Because a pressure of 500 atm is very high, we expect the gas to deviate from ideal behavior and thus eliminate choice A. When the density becomes very high (as it would here), the volume of the gas molecules themselves must be taken into account. The value of V2 calculated above would be the volume of the sample not including the molecules themselves; at typically low values of pressure and density, the additional volume of the molecules is negligible. But here, taking the non-ideality of the gas into account, the volume will be 0.02 L plus the volume occupied by the gas molecules.

26. D. Using the equation y = ½at2, with a = g, we find that y = ½(10 m/s2)(10 s)2 = 500 m. [Notice that the mass of the object is irrelevant; in the absence of air resistance (free fall), all objects fall with the same acceleration: g.]

27. D. Light travels slower through a piece of glass than through the air (this is why the index of refraction for glass—typically around 1.5—is greater than the index of refraction for air—which for all practical purposes is 1); this observation eliminates choices A and C. Now what about sound? Sound generally travels faster through liquids than through gases, and faster still through solids (because the bulk modulus—which measures resistance to compression—is so much greater in solids than in liquids or in gases). Therefore, we expect the sound wave to speed up as it enters the glass from the air.

Page 6: AAMC 7R Solutions

MCAT Practice Test 7 Solutions6

Passage V (Questions 28–32)

28. B. To determine absorbance, we use Beerʼs law (Equation 3 in the passage): A = εcl. Thus, to determine A, we need the values of ε, c, and l. The value of ε is given in the question, 198 M –1 cm–1, and the passage states that the path length l is 1 cm.To find c, we use Table 1: in Experiment 1, the concentration of Br2 (bromine, the absorbing species) is 4.14 × 10–3 M. Therefore,

A = εcl = (198 M –1 cm–1)(4.14 × 10–3 M)(1 cm) ≈ 200(4 × 10–3) = 0.8

29. A. First, the rate law given in Equation 2 can be simplified: as the last sentence of the passage states, the reaction is zero order with respect to bromine; therefore, the rate law may be expressed more simply as Rate = k[acetone]a[H+]c. Now, to determine the exponent c, letʼs compare, say, the data for Experiments 4 and 6 given in Table 1. The values of [acetone] are the same and the rate constants are approximately the same, but the value of [H+] is twice as great in Experiment 4 as in Experiment 6. Since the overall rate is also twice as great (approximately) in Experiment 4 as in Experiment 6 (12.9 × 10–6 M s–1 vs. 5.99 × 10–6 M s–1), we conclude that the rate is proportional to the first power of [H+]; that is, the value of c is 1.

30. A. The question states that the reaction is first order with respect to acetone, so choice D is eliminated; and as the last sentence of the passage states, the reaction is zero order with respect to bromine, so choices B and C are eliminated.

31. B. According to the passage, acetone has a mass of 58.0 g/mol and a density of 0.791 g/mL. Therefore, for pure acetone we find that

32. D. The regions that border the visible portion of the electromagnetic spectrum are the ultraviolet (UV) and the infrared (IR), so choices A and B are eliminated. The visible region of the EM spectrum spans wavelengths from roughly 400 nm to 750 nm (or, more precisely, 390 nm to 780 nm), so light of wavelength 395 nm is indeed in the visible region, very close to the shortest visible wavelength. Because we know that UV radiation is more energetic than visible, UV light has a shorter wavelength than visible (recall the formula E = hc/λ for the energy of a photon with wavelength λ). Therefore, light of wavelength 395 nm (which is very close to the shortest visible wavelength) is very near the ultraviolet region, not the infrared.

Passage VI (Questions 33–37)

33. C. The first sentence of the second paragraph of the passage states that cell-phone channels operate at frequencies ranging between 824 MHz and 894 MHz. This is a range of 894 MHz – 824 MHz = 70 MHz.

34. C. First, eliminate choice A: the question asks for current, not power. Write the equation for the power of a circuit element in the form I = P/V. The passage tells us that the maximum power of a cell-phone signal is P = 3 watts, so if V = 12 volts, then weʼll have I = P/V = (3 W)/(12 V) = ¼ A = 0.25 A.

35. D. From the passage, we learn that a person can hear and speak on a cell phone at the same time because the phone sends and receives voice data on two separate frequency channels. Given that there is a wide range of available frequencies, and each cell phone scans all of its channels to find the one with the highest signal intensity, it seems reasonable to conclude that each pair of phones being used to transmit a conversation operates on a unique pair of frequencies. Therefore, choice D is best. In addition, there is no support in the passage for choice A, B, or C (and in fact each is false).

36. D. According to the description given in the question, the intensity is proportional to P/r2, where P is the power used by the phone and r is the distance between the phone and the switching station. Comparing choices A and B, we eliminate B—it has the same power but a greater distance, so the intensity of the signal will necessarily be less. Comparing choices C and D, we can eliminate choice C for the same reason. So all we need to do is compare choices A and D. The value of P/r2 for choice A is 0.6/4 = 3/20 W·mi–2, but the value of P/r2 for choice D is 3/16 W·mi–2, which is larger (same numerator, smaller denominator).

Page 7: AAMC 7R Solutions

Physical Sciences Solutions 7

37. A. Sound waves through the air are longitudinal (air molecules oscillate in the direction parallel to the direction of wave propagation), but electromagnetic waves are transverse (the electric and magnetic fields oscillate perpendicular to the direction of propagation). Therefore, choice A is correct, while choices B and C are wrong. Choice D is also wrong. In general, sound waves have lower frequencies and longer wavelengths than electromagnetic waves. For example, a typical acoustic wave—such as that produced by a human voice—has a frequency around 175–350 Hz and travels through the air at a speed of about 350 m/s. Therefore, using λ = v/f, the wavelength of such a wave is roughly 1 to 2 m. But an 846 MHz electromagnetic wave has a wavelength of λ = c/f = (3 × 108 m/s)/(846 × 106 Hz), which is less than ½ m.

Passage VII (Questions 38–42)

38. C. Choice B is false, and while choices A and D are true, neither one answers the question. The answer is C. Positive charge will accumulate on the top plate of the capacitor and negative charge will accumulate on the bottom plate. As this charge accumulates, the voltage across the capacitor increases, until it matches the voltage of the battery; at this point, no additional charge can accumulate, as it is repelled by the charge already present on the plates.

39. B. The discharge cycle occurs when the capacitor is fully charged and the switch S is then closed to the right. As the capacitor loses its charge, its voltage decreases (remember that for a capacitor, V is proportional to Q, so as Q decreases, so does V). So, from Ohmʼs law, I = V/R, we can see that R must decrease as V decreases if we wish to keep I constant.

40. B. Total energy is conserved in the process of charging the capacitor. Since we know that current flowing through the resistor r causes the resistor to dissipate energy as heat, the work done by the battery is converted into potential energy of the capacitor plus some heat. Therefore, the potential energy gained by the capacitor will necessarily be less than the work done by the battery.

41. C. A charged capacitor stores electrical potential energy, and a battery stores chemical energy (which is then converted to electrical energy during the batteryʼs operation). Therefore, Items I and III are true, so the answer must be C. Resistors dissipate energy as heat when current flows through them; resistors do not store energy.

42. D. The passage states that the voltage of the battery is 12 V, and Figure 2 shows that at time t = 0, the discharge current is 0.002 A = 2 × 10–3 A. Therefore, using Ohmʼs law in the form R = V/I, we can calculate that the resistance of the variable resistor at t = 0 must have been R = V/I = (12 V)/(2 × 10–3 A) = 6000 Ω.

Passage VIII (Questions 43–48)

43. D. The passage states that the rate law for Reaction 3 is Rate = k[ClO3–][Br–][H+]2, so the rate of formation of Cl– in

Reaction 3 is proportional to [H+]2. If the pH is increased from 1 to 2, then the value of [H+] decreases by a factor of 10, so it follows that the rate of formation of Cl– would decrease by a factor of 102. Therefore, if the rate of formation of Cl– is 1 × 10–2 M/s at pH 1, then it will be (1 × 10–2)/102 = 1 × 10–4 M/s at pH 2.

44. D. This question concerns kinetics, but the only additional information the passage gives about Reactions 1 and 2 concerns thermodynamics (namely, the values of the equilibrium constants). The values of Keq tell us nothing about the rates of the reactions, so we cannot compare the rates from the information given.

45. A. The passage indicates that Reaction 1 is extremely slow, and that Reaction 2 is faster. It follows that the reactants must interact more readily in Reaction 2 than they do in Reaction 1. Increasing the repulsive forces—or reducing the attractive forces—between reactants would not allow them to interact more readily, thus eliminating choices B, C, and D.

46. A. Sequence III in the second reaction mechanism given in the passage tells us that the reaction of (unlabeled) SO3 and (labeled) H2

18O to form SO318O2– is fast, so without any additional information, choice A is best.

Page 8: AAMC 7R Solutions

MCAT Practice Test 7 Solutions8

47. C. In Reaction 2, we have HOCl reacting with NO2– to form NO3

– (and HCl), so the O–Cl bond must break and a new N–O bond must be formed. While all the choices show the O–Cl bond breaking, only choice C shows a new N–O bond forming.

48. B. The mechanism for Reaction 3 is given in the passage: Step I is fast, Step II is slow, and Step III is fast. Because II is slower than I or III, the activation energy for Reaction II must be larger than the activation energy for I or III. In terms of the energy diagram, the rise in energy from reactants to activated complex should be higher for II than for I or III. Only the diagram in B shows this behavior.

Free-Standing Questions (49–53)

49. D. Because we are asked to compare the boiling points of ammonia and phosphine, the statement in choice A is irrelevant (not to mention false). Choice B is also irrelevant—because boiling is related to the weakening of intermolecular forces, not intramolecular forces—and false besides. And choice C is backwards; we usually expect higher molecular weight compounds to have higher boiling points, not lower ones. The best answer is D: since the intermolecular forces are stronger in ammonia than in phosphine (which actually canʼt hydrogen bond), this directly implies that ammonia should have the higher boiling point.

50. C. Density is mass per volume: = m/V. If we apply this formula to each of the four objects listed in the table, we find Object A: A = mA/VA = (1.5 g)/(0.5 cm3) = 3 g/cm3

Object B: B = mB/VB = (3.0 g)/(0.75 cm3) = 4 g/cm3

Object C: C = mC/VC = (4.5 g)/(1.0 cm3) = 4.5 g/cm3

Object D: D = mD/VD = (6.0 g)/(1.5 cm3) = 4 g/cm3

Clearly, C is the highest.

51. B. Since the halogens need only one more valence electron to complete their octet configuration, these elements gain an electron readily. Chlorine (choice B) is a halogen. The other choices are Group 1 or Group 2 metals, which would be more willing to lose an electron than gain one.

52. C. NH4+, the ammonium ion, is the conjugate acid of the weak base NH3. NH4

+ is therefore a weak acid, not a base.

53. B. We use Snellʼs law, n1 sin 1 = n2 sin 2. We take n1 = 1 (for air) and weʼre given that 1 = 45° and 2 = 30°. Therefore, Snellʼs law becomes sin 45° = n2 sin 30°, so n2 = (sin 45°)/(sin 30°). Since sin 45° = 0.707 and sin 30° = 0.5, we find that n2 = (0.707)/(0.5) = 2(0.707) = 1.414, so choice B is best.

Passage IX (Questions 54–60)

54. C. According to Table 1, the solubility of Pb(NO3)2 in water at 0°C is 37.7 g/100 mL. Using the formula weight for Pb(NO3)2, which is also listed in the table, we find that the molality of a saturated Pb(NO3)2(aq) solution is

Since 377/331 is clearly a little more than 1, we know from the available choices that the answer must be C, 1.14 m.

55. C. The freezing point will be changed by iKf m, where i is the vanʼt Hoff factor of the solute, Kf is the freezing-point constant of the solvent, and m is the molality of the solution. For ethylene glycol, i = 1, and the Kf value for water is –1.86 °C/m. Therefore, the freezing point here will be lowered by (1)(1.86 °C/m)(10.75 m) ≈ (1)(2)(11) = 22°C ≈ 20°C, so the freezing point of the ethylene glycol solution will be 0°C – 20°C = –20°C.

Page 9: AAMC 7R Solutions

Physical Sciences Solutions 9

56. A. The freezing point will be lowered by iKf m, where i is the vanʼt Hoff factor of the solute, Kf is the freezing-point constant of the solvent, and m is the molality of the solution. Since both solutions in this question are aqueous and have the same concentration, the amount by which the freezing point is lowered is proportional to i. Because the value of i is 3 for lead nitrate [since Pb(NO3)2 dissociates into Pb2+ + 2 NO3

–], which is 3 times greater than the value of i for ethylene glycol (for which i = 1), the solution of lead nitrate will lower the freezing point of water by a greater extent, by threefold.

57. C. First, eliminate choices A and D: the generalization ʻthe solubility of a solute doubles for every ten–degree rise in temperature ̓cannot be said to apply to an aqueous solution of ethylene glycol, because as Table 1 shows, the solubility is infinite at 0°C and at 20°C. So, the question now is, does the ʻlike dissolves like ̓generalization apply to ethylene glycol and water? Yes, since like water, ethylene glycol (a diol, shown here) is a polar molecule, and it can form hydrogen bonds with water. Thus, the best answer is C.

58. B. First, eliminate choices C and D: osmosis certainly does apply to biological fluids, and if the fluid outside the cell is hypertonic, the concentrations are not the same on both sides of the cell wall. By definition, osmosis is the flow of solvent through a semipermeable membrane into a more concentrated solution, so if the fluid outside the cell is hypertonic, solvent will flow out of the cell. This will cause the remaining fluid inside the cell to become more concentrated.

59. B. According to Table 1, the solubility of Pb(NO3)2 in water at 0°C is 37.7 g/100 mL. Therefore, if 39.0 g of lead nitrate were added to 100 mL of water at 0°C and allowed to equilibrate, weʼd expect 37.7 g of the solute to dissolve and the remainder, 39.0 – 37.7 = 1.3 g, to be left undissolved.

60. A. An example of an aqueous solution containing 10% lead nitrate by weight would be one that contained 10 g of lead nitrate in 100 g of solution—that is, 10 g of lead nitrate in 90 g of water. Converting each of these to moles, we find that

Therefore, the mole fraction of lead nitrate in the solution would be

Passage X (Questions 61–66)

61. B. Average acceleration is defined to Δv/Δt. In Case I, the value of Δv/Δt is (–20 m/s)/(0.1 s) = –200 m/s2. In Case II, the value of Δv/Δt is (–20 m/s)/(0.25 s) = –80 m/s2. Therefore, the ratio of the average acceleration in Case II to that in Case I is (–80 m/s2)/(–200 m/s2) = 2/5 = 0.4. [As an alternate solution, notice that the value of Δv is the same in both cases; thus, the ratio of the average accelerations is simply the inverse of the ratio of the stopping times: (0.1 s)/(0.25 s) = 0.4.]

62. B. Since the two cars have equal but opposite momenta before the collision, the total momentum before the collision is zero. By conservation of total momentum, the total momentum after the collision will also be zero. Thus, we can see that the two cars will both stop after the collision, so their combined kinetic energies before the collision will be completely converted to heat and deformation energy afterwards. The kinetic energy of each car before the collision is ½mv2 = ½(1000 kg)(20 m/s)2 = 2 × 105 J, so the total kinetic energy of both cars is twice this value, namely, 4 × 105 J.

63. D. If the coefficients of friction were reduced, then more distance would be required to stop (choice A is true and thus eliminated), and slippage on curves would indeed be more likely (choice C is true and thus eliminated). The efficiency of the carʼs engine is independent of the tires, so there is no reason to doubt the validity of choice B and so it is eliminated. The answer must be D. There is no support for the claim that the air pressure would have to be higher in hard rubber tires than in soft rubber tires.

Page 10: AAMC 7R Solutions

MCAT Practice Test 7 Solutions10

64. A. By Newtonʼs Second law, force is proportional to acceleration, so the force will be reduced if the acceleration is reduced. Because acceleration, ā = Δv/Δt, is inversely proportional to the time interval, the acceleration will be reduced if the collision time is increased. Choice A is correct. This is the same reason why air bags reduce the force on vehicle occupants involved in a collision; an air bag increases the time it takes for the occupant to come to rest, which reduces the occupantʼs acceleration, thus reducing the force the person feels.

65. D. The passage states that cars A and B are initially 100 m apart, with car B ahead of car A. So, letʼs take the initial position of car A to be x0 = 0 and the initial position of car B to be x0 = 100 m. Since both cars travel at a constant speed (car A at 30 m/s, car B at 20 m/s), we can use the equation x = x0 + vt to give their positions at any time t. For car A, we have xA = 30t, and for car B we have xB = 100 + 20t. Car A overtakes car B when xA = xB—that is, when 30t = 100 + 20t. Solving this equation for t, we get t = 10 s.

66. C. Before the collision, the total momentum of the cars is pA + pB = mvA + mvB = m(vA + vB); after the inelastic collision (which we can assume is perfectly inelastic, since the question asks for the post–collision speed [singular] of both cars), the total momentum is pA&B = (2m)vA&B. So, by conservation of total momentum, it must be true that m(vA + vB) = 2mvA&B. Canceling the mʼs and dividing by 2, we get vA&B = ½(vA + vB) = ½(30 m/s + 20 m/s) = 25 m/s.

Passage XI (Questions 67–71)

67. D. Neither choice A nor choice C would cast doubt on the coincidence hypothesis, since they do not provide clear evidence for a less random explanation. Choice B could help support the coincidence hypothesis: if the distances from the main quake to the subsequent quakes were excessive, what mechanism could be responsible for transmitting the influence of the main quake to the locations of the subsequent quakes? But choice D would support an alternate hypothesis: if the subsequent quakes occurred in all possible directions around the location of the main quake, this would support the hypothesis that the main quake was the cause of the subsequent ones—just like a rock dropped into a pond, the effect of which is transmitted in all possible directions (as the waves on the surface of the pond expand from the point of impact).

68. C. The passage tells us that the size of lasting deformations falls off as (L/d)3, where d is the distance from the quake center. In other words, the lasting deformations are inversely proportional to d 3. So, if the distance increases by a factor of 4 (from 4L to 16L), then the lasting deformations decrease by a factor of 43 = 64.

69. A. Choice C says that the earth shook at nodes of a standing wave; but nodes are points of no displacement, so we eliminate C. We eliminate choice D for a similar reason: if destructive interference were maximum, there should be little or no displacement. Although both choices A and B mention local stress (which allowed the triggered quake to occur even though the energy transfer was relatively small), choice B says that these stresses were caused by surface waves (that is, waves in the earthʼs crust). But the second-to-last sentence of the third paragraph of the passage tells us that such surface waves “cause little lasting deformation,” so we conclude that A is a better answer than B.

70. B. According to the passage, surface waves have a periodicity of about 10 seconds. Since f = 1/T, this means that surface waves have a frequency of about 1/10 Hz. Therefore, if λ = 20 km = 20,000 m, the equation v = λf implies that the speed of these waves is v = λf = (20,000 m)(1/10 Hz) = 2000 m/s, so choice B is best.

71. C. As a wave source moves, the waves in front of the source get compressed, while those behind the source get expanded. (This is the Doppler Effect expressed in terms of wavelength; in terms of frequency, waves in front of the source have a higher frequency while those behind have a lower frequency.) So, if the source of the waves moves from south to north, the waves moving northward decrease in wavelength while the waves moving southward increase in wavelength.

Page 11: AAMC 7R Solutions

Physical Sciences Solutions 11

Free-Standing Questions (72–77)

72. C. The difference in hydrostatic pressure between two points in a fluid separated by a vertical distance Y is equal to gY, where is the density of the fluid and g is the acceleration due to gravity. Substituting in the given values of , g, and Y, we find that ΔP = gY = (1000 kg/m3)(10 m/s2)(0.25 m) = 2,500 N/m2.

73. B. Because the given reaction occurs spontaneously in the forward direction, the species with the highest electron affinity must be one of the reactants; this eliminates choices C and D. Now, because each hydrogen ion takes an electron from cadmium, H+(aq) must have a higher electron affinity than Cd(s).

74. D. Choices B and C are wrong since the atom does not emit or absorb an electron when one of its electrons “jumps” from one energy level (shell) to another. The reverse process of an electron moving from the n = 3 shell to the n = 2 shell would result in the emission of a photon (as given in choice A, which is therefore incorrect), so the process described in the question must involve the absorption of a photon, which increases the energy of the atom. Alternatively, we can use the formula given in the question, as follows. When the electron is in the n = 2 shell, the energy of the atom is –C/4, and when the electron is in the n = 3 shell, its energy is –C/9. Because C is a positive constant, –C/9 is greater (less negative) than –C/4, so when the electron goes from the n = 2 shell to the n = 3 shell, the energy of the atom increases.

75. B. To determine the ratio nliquid/nair, which is the relative refractive index of the liquid compared to air, we apply Snellʼs law, nair sin 1 = nliquid sin 2, and find that nliquid/nair = (sin 1)/(sin 2). So, if we know 1 and 2, we will know the value of nliquid/nair. The angle of incidence (1) is known, because the angle of reflection is known and the angle of reflection is equal to the angle of incidence. Therefore, we only need the angle of refraction, 2, in order to determine nliquid/nair.

76. A. By definition, 1 faraday is the magnitude of charge on 1 mole of electrons, so 0.1 faraday is equal to the magnitude of charge on 0.1 mole of electrons. But since it takes 3 electrons to reduce each Al3+ ion to Al, if 0.1 mole of electrons are available to accomplish this reduction, only

will be reduced.

77. D. An indicator, HIn, is a weak acid, so it participates in a proton–transfer equilibrium in aqueous solution: HIn + H2O In– + H3O

+. The indicator changes color at the end point of the titration—that is, at the point when [HIn] = [In–]. The equilibrium constant for this reaction is given by Ka = [In–][H3O

+]/[HIn], so when [HIn] = [In–], we have Ka = [H3O+]. Taking

the negative log of both sides of this equation, we see that the indicator undergoes a color change when pKa (of HIn) = pH, so choice D is best.

Page 12: AAMC 7R Solutions

MCAT Practice Test 7 Solutions12

VERBAL REASONING SOLUTIONS

Passage I (Questions 78–84)

78. CA: The author criticizes those who argue that maintaining biodiversity should be a priority (lines 11–17). Overall, the passage

suggests that biodiversity is not as important as the life historians (line 4) believe it to be. Furthermore, there is no link between biodiversity and human survival mentioned in the passage.

B: This point is made in lines 33–34, but it is too narrow to be the central thesis of the entire passage. The difference between cultural and genetic adaptation is one point made in service of the authorʼs larger argument about the nature of human influence on the world.

C: Yes. Even though this argument is explicitly made only in the last two paragraphs (and stated most directly in lines 52–56), the rest of the passage leads up to and supports it. First, the author describes and criticizes the position of life historians who believe disaster through mass extinctions is at hand (paragraphs 1 and 2). The author then introduces the argument that humans are unique in their capacity for consciousness and so for rapid cultural adaptation (paragraphs 3 and 4). This leads to the authorʼs defense of human action on the world as natural (paragraph 4) and as beneficial (paragraph 6). Choice C is not a perfect answer, but it is more inclusive than choices B and D, and overall the best of the four choices.

D: The author makes this point in the first sentence of the passage, but it is too narrow to be the central thesis or main point of the entire passage.

79. AA: Yes. In paragraphs 3 and 4, the author argues that the human facility for reason, described as “Consciousness.

Mind. Insight.” (line 23) gives us the ability to think through and rapidly adapt to new situations (lines 35–43). If some other species had an even greater capacity for reasoned judgement, we could infer that that species would have a similar (even greater) ability to adapt.

B: The author does not suggest that genetic or cultural adaptation led to the human capacity for reason; the passage does not explain how the “idea of personhood” (line 21) came into existence. Therefore, we could not infer that other animals are on an evolutionary or adaptive path towards reason, or that the existence of reason in some other species indicates a faster rate of adaptation.

C: Be careful not to use outside knowledge. The passage does not suggest that the capacity for reason plays a role in defining biological classifications; biological classification is never discussed.

D: This choice is not relevant to the new information in the question stem (we donʼt know that the existence of other reasoning animals would further erode biodiversity). Furthermore, it is inconsistent with the authorʼs implication that a decrease in biodiversity is not as tragic for the environment as some have made it out to be (lines 11–22, 44–56, and 60–64).

80. DA: The author does not suggest that the rate of extinction will decrease, either “naturally” or through human action.B: This choice is inconsistent with the tone of the passage. The author describes the belief that we must “stem the erosion of

biodiversity” or the rate of extinction largely in order to criticize that belief (lines 11–17 and 44–56).C: This choice is too extreme. In paragraph 5 the author concedes that “in some parts of the world” human beings are in fact

“trampling this planetʼs most ancient and delicate harmonies” (lines 44–48). Thus, the author might believe that human activity is playing some role in the increasing rate of extinction.

D: Yes. This choice is consistent with the tone of the passage. The author describes the opinion of those who are concerned or panicked about the increasing rate of extinction (lines 3–10), but then goes on to indicate that these ideas do not “give an adequate picture of recent biological history” (lines 15–17). At the end of the passage, the author states that the changes in the world wrought by human activity are not “making its ʻnatural ̓continuance impossible” (lines 52–56) and that the presence of human beings has not made North America “a poorer place” (lines 62–64). Overall, then, the author would say that there is no reason to panic.

81. CA: Even though the question stem introduces new information, the correct answer still must be relevant to and consistent with

the themes presented in the passage. The author argues that the need to preserve native habitats and species on Earth is not as pressing as some would have us believe (lines 11–22, 44–56, and 60–64). There is no reason to think that the authorʼs

Page 13: AAMC 7R Solutions

13Verbal Reasoning Solutions

opinion would change in a discussion of human habitation of Mars.B: As in choice A, this statement is not consistent with the focus of the passage. The author emphasizes the value and

importance of human action on the world; he never states that it is especially important to preserve other species, either for their own sake or because of their usefulness to human beings. This choice is a trap based in part on the reference to “domesticable” (line 43).

C: Yes. A main theme in the passage is the unique ability of human beings to culturally adapt to, and take advantage of, their surroundings (lines 33–43). Therefore, we can infer that the author would discuss the role of human adaptability or inventiveness in colonizing and surviving on Mars.

D: This choice is too extreme. In the passage, the author indicates that the emphasis some place on biodiversity is exaggerated (lines 15–17). However, the author does not go so far as to suggest that biodiversity is irrelevant, on Earth or elsewhere.

82. AA: Yes. In lines 3–6, the author writes, “What alarms so many life historians is not that extinctions are occurring but

that they appear to be occurring at a greater rate than they have at all but a few times in the past....” The word “rapidly” in the answer choice is a bit strong, but this is still the best choice of the four.

B: This choice is too extreme to describe the life historians ̓views. The author suggests that the historians believe human action plays a role, but not that they believe human intervention is the only cause. Note that the historians claim that extinctions are happening faster than “at all but a few times in the past” (lines 5–6). Therefore, it is possible that similar rates of extinction at times occurred before human history began.

C: The correct answer will describe evidence presented by the historians to show that human beings are in part at fault for the current extinction rate. The passage does suggest that life historians raise “the specter of the sort of wholesale die-offs that ended the reign of the dinosaurs” (lines 3–8) as a frightening analogy to what may be occurring today. However, the extinction of the dinosaurs (which happened before humans came along) is not given as evidence that we risk destroying species.

D: No analogy is drawn between humans and dinosaurs. The life historians, as described in the passage, do not claim that humans are at risk of extinction (or that the dinosaurs caused their own demise).

83. CA: Beware of using your own definition of “cultural.” By cultural adaptation, the author means the use of reason to adapt to

new circumstances (lines 35–43), not getting to know members of other societies or cultures.B: In the passage, the author describes cultural adaptation as learning how to understand and use new aspects of the environment

through the use of reason. The author also indicates that this involves changing our environment to suit our own needs (lines 51–56). While moving around to different areas could involve adapting to and manipulating different environments, we canʼt speculate about what might occur and then read that possibility into the choice as it is written. When you compare this answer to choice C, choice C explicitly includes the necessary themes.

C: Yes. In paragraphs 4 and 5, the author describes cultural adaptation as the use of reason to puzzle out new circumstances and to adapt the environment to suit our own needs.

D: As in choice A, this answer uses a different definition of “cultural” than does the author. Furthermore, the author discusses how humans have changed the environment to suit their own desires (lines 44–60), while this choice indicates that humans move around until they find an environment that already suits them as is.

84. DA: The author never raises the issue of selfishness. Furthermore, the authorʼs attitude toward human action in and on the world

is largely positive, while this choice has a negative or critical tone towards human behavior (suggesting that we must pay attention to selfish motives that guide it).

B: This choice takes a view opposite to that of the author. The author states that if we donʼt recognize the positive changes caused by human behavior, we will have too pessimistic a view of ourselves and of the future (lines 60–66).

C: This choice is inconsistent with the theme of the passage as a whole. The author is not concerned with the possibility that other species might be undervalued, but instead with the possibility of placing too little value on human beings (lines 52–64). In fact, the author appears to be concerned that we are currently placing too much importance on the existence of other species (lines 15–22 and 23–32).

D: Yes. In the final paragraph, the author writes, “The more convinced we are that our species is a plague, the more we are obliged to yearn for disasters” (lines 64–66). This means that the more we believe that change caused by human beings is largely destructive, the more likely it is that destruction or disaster will occur.

Page 14: AAMC 7R Solutions

MCAT Practice Test 7 Solutions14

Passage II (Questions 85–94)

85. BA: This choice contradicts the passage. In the second paragraph the author states that artists in the past were not successful in

their attempts to “see the world objectively” (line 24) in part because “there always intervened between the visual event and the act of realizing the vision an activity which we can only call interpretative” (lines 29–32). Thus, objectivity and interpretation are inconsistent with each other.

B: Yes. The author writes in paragraph 2 that interpretation “seemed to be made necessary by the very nature of perception” (lines 32–33). Here, perception means the evidence presented by the senses, that is, by the act of seeing.

C: This choice contradicts the passage. Cézanne, according to the author, was the first painter who determined to “see the world objectively” (line 3), whereas previous painters relied on “an activity which we can only call interpretative” (lines 31–32).

D: This choice reverses the chronology presented in the passage. The author cites previous periods of art (Greek, Roman, and Renaissance) when painters relied on interpretation (lines 24–29). Cézanne founded “the modern movement in art” which attempted to avoid interpretation and to present the world “as an object, without any intervention either of the tidy mind or the untidy emotions” (lines 6–7).

86. CA: This choice is too narrow to be the central thesis. The author makes this statement in the first paragraph (lines 8–14) in

service of a larger argument about the distinct and revolutionary nature of Cézanneʼs ideas.B: Perspective and interpretation are presented as complementary. In the third paragraph, the author lists perspective (line

48) as part of the use of intellect, an “extra-visual [faculty]” (lines 42–43). This faculty is used by artists to interpret rather than to objectively portray reality. Furthermore, the passage discusses perspective and interpretation in order to discuss the revolutionary aspect of Cézanneʼs ideas.

C: Yes. The author begins by stating that Cézanne wished to see the world objectively, “without any intervention either of the tidy mind or the untidy emotions” (lines 6–7). In the rest of the passage, the author describes how Cézanne, unlike previous artists, attempted to avoid the intervention of interpretation (paragraph 2), that is, the use of imagination or intellect (paragraph 3). The last paragraph sums up how this effort distinguished Cézanne from his predecessors.

D: While this is true according to the passage (lines 24–29), it is too narrow to be the central thesis. The author makes this statement in order to contrast those artists with Cézanne (lines 56–61); the main focus of the passage is on Cézanne himself.

87. AA: Yes. In the second paragraph, the author writes, “Great revolutionary leaders are people with a single and a simple

idea, and it is the very persistency with which they pursue this idea which endows it with power” (lines 19–21).B: The author states that part of Cézanneʼs revolution was his attempt to represent the world objectively by eliminating

interpretation from his art (lines 1–7, 14–18, and 56–61).C: The passage describes how artists before Cézanne tried (unsuccessfully) to “represent the world ʻas it really is ̓” (lines

24–29). However, these artists are not described as revolutionary. It is Cézanne, who tried to “penetrate to the reality that did not change” (lines 15–16) by “ ʻrealizing ̓his sensations in the presence of nature” (lines 60–61), who is portrayed as a revolutionary (lines 1–7, 19–24, and 56–61). Yet the author states that “revolutionary leaders are people with a single and a simple idea” who doggedly pursue that idea (lines 19–21). Thus, it is the pursuit of a single idea, not the pursuit of “realism” itself that defines a revolutionary.

D: Cézanne, described by the author as a revolutionary, tried to realize “his sensations in the presence of nature” (lines 60–61), not to distinguish between nature and art. In fact, he tried to eliminate any distinction or difference between nature and art by showing “the reality that did not change” beneath the “shimmering and ambiguous surface of things” (lines 14–16).

Page 15: AAMC 7R Solutions

15Verbal Reasoning Solutions

88. BA: Note the words “without intervention” in this answer choice; the author implies just the opposite. The use or intervention

of intellect and imagination involves interpretation (lines 41–52), and interpretation is consistent with subjectivity.B: Yes. In the first paragraph, the author writes that the Impressionists “had seen the word subjectively—that is to say,

as it presented itself to their senses in various lights, or from various points of view” (lines 8–11).C: While a subjective portrayal (or view) may rely on imagination to “create an ideal space occupied by ideal forms” (lines

44–45), this is not given as the definition of subjectivity. That is, while viewing or presenting an object within an ideal space entails subjectivity, this does not mean that all subjective views involve ideal space. See the solution for choice B; the passage gives a specific definition of subjectivity in lines 8–11.

D: According to the passage, both subjective and objective views represent objects and/or nature. Objects and nature are not distinguished from each other, nor does either represent a distinction between subjective and objective approaches. This choice is a trap based in part on the last line of the passage, where the author writes that Cézanne (who strove for objectivity, not subjectivity) attempted “ ʻrealizing ̓his sensations in the presence of nature” (lines 60–61).

89. DA: Notice the word “not” at the end of the question stem. This answer choice is the opposite of what the author suggests. In

lines 25–29, the author states that before Cézanne “there have been attempts to make art ʻimitative ̓” and to “represent the world ʻas it really is. ̓”

B: This choice is out of scope. While Impressionism is discussed in the first paragraph, the author never discusses what artists before Cézanne thought about its value.

C: The passage is about artists ̓attempts to represent reality in their art. The author states that interpretation “always intervened between the visual event and the act of realizing the vision” (lines 29–31). Therefore, while artists before Cézanne may have been unable to portray reality, the author suggests that they could in fact see or perceive it.

D: Yes. The second and third paragraphs discuss how artists before Cézanne either did not wish to or were unable to represent reality. That is, they failed to portray “the world ʻas it really is ̓” (line 29) or “what the eye sees” (line 49).

90. BA: While the author states that in the Greek, Roman, and Renaissance periods artists attempted to make art “imitative” (lines

24–29), the passage does not discuss or describe specific works of art.B: Yes. See the solution for choice A. The author does not describe any specific works of art from these periods (see

paragraph 2).C: While the nature of perception is discussed in lines 32–40, this is in support of the claim that Greek, Roman, and Renaissance

art was not able to represent the world as it is, not in support of the claim that it tried to do so.D: There is no later evidence that contradicts the claim that such art tried to represent the world as it is.

91. AA: Yes. The author states that “the modern movement in art begins with the single-minded determination of a

French painter [Cézanne] to see the world objectively” (lines 1–3). In the second paragraph, the author states that revolutionary leaders pursue “a single and a simple idea, and it is the very persistency with which they pursue this idea that endows it with power” (lines 19–21).

B: This choice is inconsistent with the passageʼs description of Cézanneʼs pursuit of objectivity. Cézanne strove to avoid the use of “extra-visual faculties” such as imagination and intellect (lines 5–7 and 56–61). Even if he failed to do so, it would not be his (inadvertent or unwilling) use of such faculties that gave his work power.

C: As in choice B, this contradicts the passage. Cézanne sought to eliminate the use of interpretation, not to expand upon the concept (lines 5–7, 14–18, and 56–61).

D: First, Cézanne sought to “penetrate to the reality that did not change” and not to paint “the bright but deceptive picture presented by the kaleidoscope of the senses” (lines 15–18). Second, the author specifically indicates that it was Cézanneʼs dedication to pursuing the idea of objectivity that made his work influential (lines 19–21).

Page 16: AAMC 7R Solutions

MCAT Practice Test 7 Solutions16

92. BA: The passage does not argue that Cézanne pursued similar goals. While Greek and Roman art was “imitative” and tried to

“represent the world “as it really is” (lines 24–29), the author states that Cézanne was the first to attempt to “see the world objectively” (lines 1–7). Furthermore, even if the author had made the claim that Cézanne pursued similar goals, the new information in the question stem would be consistent, not inconsistent with that claim.

B: Yes. The author claims in the first paragraph that “what we call the modern movement in art begins with the…determination of a French painter [Cézanne] to see the world objectively” (lines 1–3). If Cézanne learned the concept of objective painting from another artist, it would weaken this claim by indicating that modern art did not begin with Cézanne.

C: If Cézanne had learned objective painting from another artist, it would be consistent, not inconsistent with the statement that Cézanne respected other artists ̓(including his predecessorsʼ) attempts to represent nature.

D: This choice is essentially the same as choice C. The “art of the museums” is the art of Cézanneʼs predecessors (lines 56–58). If Cézanne learned from other artists, it would be consistent with the statement that he was familiar with the art made by his predecessors.

93. CA: The passage does not describe or analyze specific paintings. See also the explanation for question 90, choice A. B: The question focuses on artists before Cézanne. The fact that Cézanne himself rejected interpretation does not support the

claim that previous artists failed to do so.C: Yes. At the end of the second paragraph the author states that “the very nature of perception” seemed to require

interpretation in order to translate visual sensations onto the canvas. The author then goes on in the third paragraph to argue that artists before Cézanne relied on “extra-visual faculties” such as imagination and intellect in order to “solve such problems” (lines 41–48). Finally, the author states that these extra-visual faculties (for example, the use of perspective) still do not give us a picture of underlying and unchanging reality (lines 51–52) and so also require interpretation.

D: The author does not describe Cézanneʼs working methods. Note that the question asks about artists before Cézanne; an answer choice that focuses solely on Cézanne himself is unlikely to be correct.

94. DNote: Lines 48–51 tell us that “a system of perspective is no more an accurate representation of what the eye sees than a Mercatorʼs projection….” The next sentence begins, “Like the map, it serves to guide the intellect….” Therefore, we do not need outside knowledge in order to infer that a Mercator projection is a map.A: This is inconsistent with the passage. When artists use perspective to try to represent the world realistically, they rely on

the intellect as a guide. This is just one representation of reality out of many possible views. (Note the similarity of this argument to the authorʼs description of the Impressionists, who showed the world “in various lights, or from various points of view” [lines 10–11].) The Mercator projection is also just one representation, a view or perspective that is no more accurate than the view of Earth from Sirius. Therefore, maps fail as well.

B: The author makes no such comparison. The author does not describe improvements or evolution in the use of perspective, nor does he suggest that mapmaking has not evolved to become more sophisticated. Choices A and B have the same problem; the author draws an analogy between perspective and mapmaking, rather than suggesting that one is better than the other.

C: The author compares perspective and mapmaking in order to argue that both fail to show the reality beneath objects (lines 51–52).

D: Yes. The author argues that mapmaking and the use of perspective are similar in that both rely on the intellect to construct one version (not the underlying truth) of “what the eye sees” (lines 41–52). See in particular lines 51–52 where the author states, “Like the [Mercator] map, it serves to guide the intellect; perspective does not give us any glimpse of the reality.”

Page 17: AAMC 7R Solutions

17Verbal Reasoning Solutions

Passage III (Questions 95–101)

95. CNote: The language of the author and the tone of the passage are clearly non-scientific. The author discusses nature in spiritual or poetic terms, using images meant to invoke feelings and sensations rather than to provide scientific explanation or analysis. Therefore, any choice that represents the author as a scientist will be incorrect. This by itself is enough to eliminate choices A, B, and D.A: While the author does discuss the beliefs of her ancestors, “the people who came before me” (who may or may not be

“primitive”) in the last paragraph, an anthropologist would analyze those beliefs in a more scientific way (such as their role within the structure of primitive society). Furthermore, the focus of the passage as a whole is on nature itself, not on peopleʼs beliefs about nature.

B: While the author does discuss the interactive balance among species, it is not from the perspective of an ecologist. For example, an ecologist would be unlikely to describe the coordinated flowering of bamboo plants (lines 36–42) as arising from a mysterious summons, or from “some current we cannot explain” or “communal knowledge.” Rather, an ecologist would seek to explain the biological mechanisms that cause these plants to flower simultaneously.

C: Yes. The author describes the cycle of life through the changing seasons in which, for example, the death of a sunflower leads to the birth of other flowers in the future (lines 22–24). The author also describes how one kind of bamboo blooms once every 100 years (lines 36–40) and the celestial cycle of the sun observed by ancient peoples (lines 51–57), in which a certain angle of light recurs once a year. The author uses poetic images and language to communicate these ideas. Some examples of this language are: “a drama of need and survival” (lines 25–26), “the sunflowerʼs golden language” (line 28), “the language of the earth” (line 43), “I felt something like a heartbeat, a hardly perceptible current that stirred a kinship and longing in me, a dream barely remembered” (lines 45–48), and “You are the result of the love of thousands” (line 60).

D: The author mentions genes only once (line 31); overall, the author describes what guides plant and animal behavior as “a certain knowing”(lines 30–31) or “communal knowledge” (lines 41–42) and “the language of the earth” (line 43). This is not the language that a biologist would use.

96. BNote: Choices A, C, and D all give scientific explanations that are inconsistent with the personal, poetic tone and language of the passage.A: While the author does describe the “drama of need and survival” enacted on and around the sunflower (lines 25–26), she

does not directly address the issue of competition for resources among similar species or suggest that such competition drives the regularity of biological cycles.

B: Yes. In paragraphs 4, 5, and 6 the author suggests that some common language or mode of communication drives the cycle of nature. For example, the author describes bamboo plants that flower at the same time once a century, and states that a “summons” or “current we cannot explain” and “communal knowledge” underlies the regularity of this cycle. The author also refers to “the sunflowerʼs golden language,” and states that an “old voice from gene or cell taught the plant to oppose the pull of gravity and find its way upward, to open” (lines 27–30).

C: While one could say that the sunflower is an environment at certain times for certain insects, the author does not say that these insects, or any other species mentioned in the passage, has adapted to exist in one unique niche in its environment. In fact, we find a certain species of bamboo in such disparate environments as the jungles of Malaysia and Pennsylvanian gardens (lines 38–39). Finally, the author describes biological cycles as guided by some form of communication, not by adaptation.

D: The author does describe changes through the seasons (or at least within one season [line 25]) but does not indicate that it is sensitivity to and detection of environmental changes that drives biological cycles. Be careful not to use outside knowledge; the author gives no particular biological mechanism as the cause, but rather describes some kind of “communal knowledge” and “mysterious…summons” as the way in which biological changes or cycles are coordinated (lines 34–42).

Page 18: AAMC 7R Solutions

MCAT Practice Test 7 Solutions18

97. DA: This choice is too extreme. While the author does indicate that some things are beyond her comprehension (for example,

“the sunflowerʼs golden language” [lines 27–28]), she does not suggest that the meaning of the death of a horse and of a sandstorm are as mysterious as this unknown language. In fact, the author describes how the sandstorm plays a role in the cycle of nature by blowing away the sunflowerʼs dead petals, after which “the birds arrived to carry the seeds to the future” (lines 22–24).

B: This choice is too negative to be supported by the overall positive tone of the passage. While the author does call the sight of the dead horse “disturbing” (line 19), she presents this as just another change “in the greater world of the plant,” not as something that mars natureʼs beauty. The sandstorm, while inconvenient for the author (lines 20–22), is also described as part of nature, not as ugly or as a blemish upon the world.

C: As in choice B, this answer choice has a strongly negative tone that is inconsistent with the passage. The author suggests that death and destruction are a part of life (for example, the sandstorm plays a role in allowing the sunflowerʼs seeds to be disseminated [lines 20–24]), not an indication of the cruelty of nature.

D: Yes. In paragraphs 3 and 4 the author describes the dead horse and the sandstorm as part of “the greater world of the plant” and so as players in “a drama of need and survival.” As part of this drama, “there was escape, exhaustion, and death” (lines 25–27). Therefore, the passage indicates that death and destruction, represented by the dead horse and the storm, are necessary parts of the process and cycle of life.

98. DNote: This is an EXCEPT/LEAST/NOT question, for which you must eliminate the choices that are supported by the passage, and choose the one that is not supported.A: For example, the author states that, “As an outsider, I never learned the sunflowerʼs golden language” (lines 27–28). The

author also describes the “summons” that causes bamboo to flower in synchrony as “mysterious,” and as a “current we cannot explain” (lines 36–42). Therefore, humans do lack the ability to detect some intraspecies messages.

B: In the last paragraph, the author describes “the people who came before me” and their attempts to observe and make sense of the world.

C: The passage suggests that, even though humans may be “outsiders,” they can in fact exist in harmony other species. The author tells of standing in a forest and feeling “something like a heartbeat, a hardly perceptible current that stirred a kinship and longing in me, a dream barely remembered” (lines 43–48). The author also describes the voices of the ancestor counseling, “Be still. Watch and listen. You are the result of the love of thousands” (lines 59–60).

D: Yes. The author indicates that, as outsiders, humans cannot not fully understand the language of nature (lines 27–28 and 40–42). It is possible then that humans will be unable to fully satisfy their curiosity.

99. BA: There is little mention in the passage of human land use; the only direct reference is to a “suburban garden in Pennsylvania”

(line 39). In that case, the author describes how a certain type of bamboo continues to flower on its 100-year cycle, just like the plants in the jungles of Malaysia. Elsewhere in the passage, the lives of plants and animals seem to carry on with no intervention from human beings. Therefore we can infer that plants and animals, native or nonnative, are not dependent on humans for their survival.

B: Yes. See also the explanation for choice A. Human land use is never directly discussed in the context of native plants and animals (if we can assume that the bamboo discussed in paragraph 5 is not native to suburban Pennsylvania). However, given that throughout the passage the author discusses the symbiotic relationship between plants, and between plants and animals, with no reference to harm or disruption caused by human activity, we can reasonably infer that human land use does not affect that symbiotic relationship. Be careful not to use outside knowledge or opinion; the author never expresses a negative attitude towards human impact on the environment.

C: See also the solution for choice B. While this statement may be true in the real world, it is not supported by the passage. While the author describes humans as something of an outsider (lines 27–28), she never suggests that we actually disrupt or do harm to the environment. The theme of the passage as a whole is one of harmony and kinship, including that between humans and nature.

D: While the author mentions bamboo in Pennsylvanian gardens (bamboo which is likely to be nonnative, given that it also exists in Malaysian jungles [lines 36–40]), she does not suggest that the bamboo displaced native species. (Be careful not to use outside knowledge. In reality, this is true of some types of bamboo when they spread beyond the garden, but the author never mentions this problem.) There is no other reference in the passage to artificially introduced species.

Page 19: AAMC 7R Solutions

19Verbal Reasoning Solutions

100. CA: The passage portrays the author as awed by the natural processes she observes in the world. The author also describes

her desire to “Watch and listen” (lines 59–60) and learn from plants and animals and the symbiotic relationships between them. Therefore, this answer choice reverses the relationship between humans and other animals suggested by the passage; animals (as part of nature) are benefactors, granting insight and knowledge to humans (recipients).

B: See also the solutions for question 95. The author, representing the role of humans in general, does not write as a scientist, but instead gives us personal stories and emotive images that show how all of life is intertwined. Therefore, the author does not suggest that humans should study nature as an object as does a scientist, but instead that we should receive its message and become as much a part of it as we can.

C: Yes. The author suggests that there are things in nature (like “the certain knowing” that guides the birds [lines 30–31]) that we do not fully understand, and expresses a desire to “Watch and listen” to nature (lines 59–60), and to learn what she can from it, as did our ancestors.

D: As in choice A, this answer choice reverses the relationship between humans and other animals as it is portrayed in the passage. The author suggests we should learn from and be guided by nature (as the author wishes for herself). If humans were seen as the parents, this would imply that we know more than nature and that we should exert some measure of control over other animals.

101. AA: Yes. In lines 27–45, the author refers to a language or method of communication that connects and guides other

forms of life. In paragraph 6, the passage indicates that this “hardly perceptible current” is this same kind of communication using “the language of the earth,” like a (life sustaining) “heartbeat.” The author senses it, if only faintly, and so feels “kinship” with or connection to the rest of nature.

B: This choice imposes a much too scientific explanation that has no support in the passage.C: The message comes from natural (“water, trees…mosses” [lines 43–45]), not supernatural sources.D: The “hardly perceptible current” comes through “the language of the earth” (lines 43–45), and connects the author to the

rest of nature. Therefore, it is a communication from other species, not a desire to communicate with other humans.

Passage IV (Questions 102–106)Note: This passage includes quite a bit of technical language and will be difficult for most people to understand. However, the central thesis alone—that atonal music has fewer rules than traditional chromatic music—will lead you to the credited response in a majority of the questions. Here are some of the phrases and sentences describing atonal music that tell us that it has fewer rules than chromatic music: “extended the range of tonal relationships to the point at which the traditional articulative procedures were no longer adequate” (lines 6–9), “based upon a rejection of any general principles regulating simultaneity and progression” (lines 10–12), “this ultimate expansion of possible relations” (lines 14–15), “take for granted nothing except the existence of a given limiting sound world” (lines 22–23), “absence of a priori functional connections among the twelve notes of the semitonal scale” (lines 27–28), “denying a priori functional precedence to any one [note]” (lines 40–41), and “no attribute other than that represented by the pitch-class name of what is informally called a note…is defined by this referential permutation of the semitonal scale” (lines 46–49).

102. DA: Rejecting regulating principles would not lead one to reject chaotic progressions. Schöenbergʼs music is not itself chaotic

(lines 50–51); however, it is less regulated and orderly than chromatic music.B: This choice is the opposite of what the author states in lines 5–7, when he writes, “The expanded harmonic vocabulary of

late nineteenth-century music had extended the range of tonal relationships” and that the “final step in this development was taken by Arnold Schöenberg” (lines 9–10). Therefore, Schöenberg embraced rather than rejected a broadened range.

C: The passage distinguishes between “chromatic” music and the use of the chromatic scale (lines 1–5). Atonality rejects “chromaticism,” but still uses “the twelve notes of the chromatic scale” [lines 1–2]).

D: Yes. The author states that Schöenberg rejected, among other things, “any general principles regulating simultaneity and progression” (lines 10–12). “Simultaneity and progression” refer to the way in which music moves from one note or tonality to another.

Page 20: AAMC 7R Solutions

MCAT Practice Test 7 Solutions20

103. CA: The passage indicates just the opposite. The author writes that it is the diatonic system that “take[s] for granted the

existence of certain properties of that system,” that is, rules (lines 18–21), while the atonal composer “can take for granted nothing except the existence of a given limiting sound world, the semitonal scale” (lines 22–24). The author also states that in atonality there is an “absence of a priori [intrinsic] connections among the twelve notes” (lines 26–27), and that it is not “reducible to a set of foundational assumptions” (lines 29–31).

B: This is true of atonal music, not the diatonic system. The author states that in Schöenbergʼs system, an “unambiguous ordering is assumed; but the degree to which this ordering actually determines the general musical procedures varies greatly from one work to another, even though they may be by the same composer” (lines 50–54). In contrast, diatonic music composers are limited by certain rules and assumptions (lines 18–21).

C: Yes. The author states that the diatonic, unlike the atonal system, “may take for granted the existence of specific properties of that system: a seven-tone scale, triadic harmonic structure, a key center, and so forth” (lines 19–21). You donʼt need to know what these things are in order to infer that they represent certain rules or guidelines telling composers how to organize or structure their music.

D: Atonal music uses the “twelve notes of the semitonal scale” (lines 24–28), while it is diatonic music that uses “a seven-tone scale” (lines 18–21).

104. AA: Yes. This choice is essentially given by the information in the question stem, once we realize that the twelve-tone

scale (atonal music) is contrasted in the passage with the traditional diatonic musical scale. If twelve-tone composers are rejecting traditional diatonic scales, then it must be true that composers may accept or reject those scales.

B: The author does not criticize the traditional scale; the fact that some composers reject it is not enough upon which to conclude that it is inadequate or flawed. There is no evidence in the passage that anyone, including the author, believed that it was needlessly repetitive; repetitiveness is never mentioned.

C: The main theme of the passage is the fundamental difference between the two systems. The statement that some composers rejected one for the other certainly does not suggest that the two are more similar than different.

D: The passage does not suggest that traditional compositions are better. In fact, the author never raises the issue of quality.

105. BA: The author never discusses how harmonious each type of music sounds. If anything, atonal music is likely to sound less

harmonious because it breaks with the traditional rules of harmony.B: Yes. If atonal compositions follow fewer rules, it makes sense to conclude that their structure would be less obvious

to listeners. The author states in particular that atonal music does not follow “a ʻsystem ̓of composition” (lines 28–33). If, on the other hand, music did follow certain rules, then by knowing the rules we would also know a lot about the basic organization or structure of that music.

C: Compare this choice to question 103, choice D. Both choices mix up diatonic and atonal music; it is diatonic music that is based on the seven-note scale (lines 18–21).

D: This choice contradicts both question 102, choice D (the correct answer) and the summary of information from the passage given in the stem of question 104. Atonal compositions deny “a priori [intrinsic] functional precedence to any one [note]” (lines 38–41).

106. AA: Yes. The author states that an “unambiguous ordering is assumed, but the degree to which this ordering actually

determines the musical procedures varies greatly from one work to another, even though they may be by the same composer” (lines 50–54).

B: This choice is too extreme. The author states that “an unambiguous ordering is assumed” (lines 50–51) and that “the premise of an ordered arrangement of the twelve notes, if it is to have any meaning, must somehow govern the essential musical events in a consistent and logical manner” (lines 57–60).

C: The author does not directly discuss the working methods of twelve-tone composers. However, by stating that the way in which the ordering of the notes determines the composition may vary between different works by the same composer (lines 50–54), the author suggests that their methods may shift, rather than remain consistent from one composition to another.

D: This choice contradicts the passage. The author states, “Neither register, duration, timbre, or intensity…is defined by” the use of the semitonal scale in atonal music (lines 45–50).

Page 21: AAMC 7R Solutions

21Verbal Reasoning Solutions

Passage V (Questions 107–116)

107. BA: The author is concerned with the ability of nations, not firms, to compete in a global market. In the first paragraph the

author states that the “low road” approach, “if the norm, must limit a nationʼs economic competitiveness, living standard, and income equity” (lines 5–10). In paragraph 6, the author applies this concern specifically to the United States by arguing that, “For the United States to compete in a eventual global economy based on skilled workers and quality products, additional employer investment in training [the high road] is needed now” (lines 57–60).

B: Yes. In the first paragraph the author speaks in broader terms, stating that the “low road” approach, “if the norm, must limit a nationʼs economic competitiveness, living standard, and income equity” (lines 5–10). However, in paragraphs 2 and 3, the author points out that only 20% of U.S. workers are on the high road (lines 20–21), while 40% are limited to the low road (lines 22–25). Finally, the author calls for changes in the U.S. economy and educational system, arguing that, “For the United States to compete in a eventual global economy based on skilled workers and quality products, additional employer investment in training [or the high road] is needed now. Policies at all levels should encourage the coordination of employer-provided training and broader schooling” (lines 57–62). Since the passage as a whole focuses on concerns and prescriptions specific to the U.S., we can conclude that the author is particularly concerned about the competitiveness of the United States.

C: The author does not focus specifically on the concerns of technology-based economies; there is no discussion of, or contrast between, economies that are more or less technological.

D: The author never discusses less-developed nations; the focus of the passage is on the United States.

108. CA: Nothing in the passage or in the new information in the question stem indicates that investors would not care about product

quality. In fact, given that the passage lists “the quality and variety of products” (line 34) as one important factor in the global market, it seems likely that investors would in fact care about product quality.

B: Nothing in the new information in the question stem or in the passage tells us that those who invest in a high-road firm would question the personnel policies (investment in attracting and maintaining highly skilled workers) that make it a high-road firm. The author herself is clearly in favor of the high-road approach.

C: Yes. This is not a great choice, but it is the best supported of the four. The author states in paragraph 2 that “only dominant firms can afford to commit resources to training and keeping employees by providing full benefits with high wages” (lines 14–17). Furthermore, in paragraph 4, the author asserts that “The high road is not an easy course for employers to take” (lines 31–32). Given that this is a recently founded small firm, it is reasonable to infer that it will take some time, given the high level of investment in labor required by the high road, for them to generate a significant profit.

D: This choice is too strong; it makes a prediction for which there is not enough support in the passage or in the question stem. Compare this choice to choice C; while it is reasonable to infer that the firm may struggle for awhile, we have no reason to believe that it will not eventually attain competitive success.

109. DNote: The credited response will be undermine the authorʼs argument in the passage. The author states that computers help firms meet the “complex requirements” of the global market (lines 36–39), in part through improved quality control, and that the ability to use computers increases the earning potential of employees (lines 52–56). Note also that the question specifically asks about the influence of computers in the workplace. Therefore, the best answer will weaken the reported relationship between computer skills and what employees do or do not earn in the workplace.A: The author suggests that employees may require training in order to be able to use computers. However, the author does not

suggest that training is required to use what is generated by computers. Therefore, the finding that less training is required to follow computer-generated schedules than to devise oneʼs own schedule is not inconsistent with the passage.

B: The author does not argue that any use of computers or technology will by itself increase a firmʼs business. Therefore, the finding that customers may respond better to a personal letter than to E-mail is not enough to undermine the authorʼs argument that computer-based methods are necessary in order to compete in a global market. Secondly, this choice is about the influence of computers on a firmʼs competitiveness in the market, not about the influence of computers in the workplace itself.

Page 22: AAMC 7R Solutions

MCAT Practice Test 7 Solutions22

C: The author does not argue that computers increase efficiency. Furthermore, no connection between automation and computer usage (i.e., that automation necessarily involves computers) is given by the passage or by the question. Therefore, this answer choice has no direct relevance to the passage or to the new information in the question stem.

D: Yes. The author argues that “Workers who use computers on the job also earn more than do those of the same education level who do not use computers at work” (lines 52–54). If mechanics using computerized methods earned less than mechanics using traditional methods, it would undermine this part of the authorʼs argument.

110. BA: While the author mentions regulations that protect certain firms (lines 17–20), there is no suggestion that these regulations

are unfair or counterproductive in any way.B: Yes. Throughout the passage, the author stresses the value of the high-road approach, and within that the need for

well educated and highly trained employees in order to make the high road possible. In particular, in paragraph 6 the author states, “For the United States to compete in an eventual global economy based on skilled workers and quality products, additional employer investment in training is needed now. Policies at all levels should encourage the coordination of employer-provided training and broader schooling.”

C: The author states in lines 22–23 that “About 40 percent of U.S. workers receive no formal training beyond a high-school education,” and that these workers are thus relegated to low-road jobs. Therefore, the problem lies in the lack of further education and training after high school, not in insufficient vocational training during high school.

D: While the author does discuss the advantages, to both firm and worker, of computer usage in industrial production, she does not explicitly advocate it. Compare this choice to choice B, which is much more strongly supported by the passage.

111. AA: Yes. In paragraph 3 the author describes workers on the middle-road as “getting some advanced education or job-

related training but unlikely to enter the dynamic high-road labor market and attract employers who would train them thoroughly to join their core workers” (lines 27–30). The author contrasts these workers on one hand with low-road workers who have no post-high-school training (lines 22–25) and on the other with high-road employees who have a depth of education and training that allows then to “react quickly to changing technologies and markets” (lines 11–14). A person with limited job training would then fit into the “middle road” category.

B: See the solution for choice A and lines 66–70. A worker with the ability and flexibility to switch to unfamiliar procedures would be a high-road, not a middle-road employee.

C: Someone who returned to college to upgrade their qualifications is more likely to be a high-road worker. This choice does not specifically correspond to the authorʼs description of the “muddy middle road” (lines 25–30).

D: The passage discusses different types of employees, not reasons why people may be chronically unemployed. Furthermore, someone who was unable to get or hold a job for any length of time because of lack of education comes closest to the low-road employees described in the passage, not the “muddy middle road” (lines 25–30).

112. BA: The focus of the passage is on training and education. The author mentions governmental regulation only once (lines

19–20), and that is in the context of regulations that protect some firms from competition. Compare this to choice B, which falls more within the scope of the passage.

B: Yes. Although the issue of an agreement to repay tuition is new, the choice as a whole is most consistent with the authorʼs call for increased investment in the training of workers.

C: The high road calls for firms to invest in their own workers (lines 11–17), not to benefit from the training provided by other firms.

D: The credited response needs to focus on training; in the passage, the author calls on more firms to take the high-road approach by investing in the education and training of their own workers (lines 57–62). Cutting costs elsewhere is not within the scope of this discussion.

Page 23: AAMC 7R Solutions

23Verbal Reasoning Solutions

113. AA: Yes. In paragraph 4, the author provides the example of quality control, and states that skilled workers who

“demonstrate expertise and responsibility” (line 43) can use computerized systems to check quality at all stages of production. If workers used a computer bulletin board to share quality-control tips, it would support the authorʼs argument that increased involvement of workers in the production process both requires computers and increases quality.

B: The author describes workers who are more independent, in the sense that they have the “expertise and responsibility” (lines 39–43) to monitor their own work. This choice is inconsistent with that description in the passage.

C: While this choice describes increased involvement, it is not in the production process, as specified by the question. Furthermore, the question also asks what would promote the outcome attributed to increased involvement. The passage describes this outcome as improved quality control (lines 39–43); we have no indication that worker participation in hiring would contribute to quality.

D: While this practice might improve quality, it is not specific to increased worker involvement in the production process; as in choice C, this is a different kind of involvement.

114. DA: The author concludes that occupational preparation and access to training are “complementary in their effect on earnings”

(lines 46–48). The new information in the question does not tell us that these former low-level employees moved up (and likely were paid more) because of training, only that they received more training once they moved up. Furthermore, the author discusses how formal training will cause a worker to be paid better than similar workers who do not get such training (lines 49–52); she does not compare the earnings of workers at different levels in this context. Therefore, the new information does not support this particular conclusion made in the passage and presented in this answer choice.

B: There is nothing in the new information presented in the question that speaks to the issue of power sharing, and nothing that is inconsistent with any part of the passage (the author does not argue that power sharing means that all workers, including management, are equal).

C: We do not know from the question stem that this is a low- or middle-road firm. If anything, a firm in which workers have the ability and opportunity to move into managerial positions is more likely to be a high-road firm. Furthermore, to call increased access to training and equipment given to managers an “inequity” would be too negative a statement to be supported by the wording of the passage.

D: Yes. Remember that you are looking for the best choice of the four, not necessarily an ideal answer. While the author does not discuss employees who hold managerial positions, she does state that in general the more training and technological expertise one has, the more one will be paid (lines 46–56). Therefore, if newly appointed managers received more training and access to sophisticated equipment, we could reasonably infer that they would be paid more, and so that there would be a disparity in earnings between management and workers.

115. DA: The author suggests instead that investment in training increases costs. This is why only “dominant firms can afford to

commit resources to training and keeping employees by providing full benefits with high wages” (lines 14–17). The low-road firms, on the other hand, minimize their investment in worker training in order to keep labor costs low (lines 5–8).

B: The author indicates that while on-the-job training is quite valuable, “specific training degrades rapidly, and narrow skills seldom transfer well to new job requirements” (lines 68–70). Therefore, the author calls for a “coordination of employer-based training and broader schooling” (lines 61–62). In highly paid work, then, where employees are most likely to be called upon to perform a wide range of tasks and to take on a certain amount of responsibility, on-the-job training would not fully compensate for other educational deficiencies. Finally, this choice says nothing to the issue of how training does not create high-wage jobs mentioned in the second part of the question.

C: This choice is too extreme. While a worker would be even better off with a combination of job-based and comprehensive training, the author clearly indicates that employer-provided training is itself of great value (lines 57–60). Furthermore, the lack of comprehensive education is not presented as a reason why training does not create high-wage jobs.

D: Yes. This statement is directly supported by the passage. In lines 75–78, the author states, “If investment in workers outpaces the number of good jobs, many very competent workers will face an employment market of many very undemanding jobs.” That is, if the jobs donʼt exist, then even with training, workers will not be able to get high (or higher) paying jobs.

Page 24: AAMC 7R Solutions

MCAT Practice Test 7 Solutions24

116. CA: The key issue in the question stem is that low-road workers make up a lower percentage of total workers (40%) than the

percentage of total firms represented by low-road firms (50%). Given that there is no evidence that the percentage of firms that are low-road is decreasing over time, this distinction does not give us any reason to think that low-road firms are more likely to fail.

B: If anything, the new information indicates just the opposite. If low-road firms make up a higher percentage of the total than do workers for those firms, then low-road firms must hire fewer workers.

C: Yes. If low-road firms make up 50% of total firms but account for a lower percentage (40%) of total workers, each load-road firm must on average hire fewer workers than each non-low-road firm. (For example, if we had 50 low-road and 50 middle- or high-road firms, then low-road firms would make up 50% of the total number of firms. If those low-road firms employed on average 10 people each, and the non-low-road firms employed on average 20 people each, the total employed by low-road firms would be 500 people, or 33 ¹/³% of the total of 1500 people employed by both low- and non-low-road firms.) Note also that the author of the passage states that low-road firms are likely to resort to outsourcing and downsizing in order to cut costs (lines 5–8); this is consistent with the conclusion that such firms would employ fewer workers.

D: These statistics about U.S. firms tell us nothing about firms outside the United States.

Passage VI (Questions 117–121)

117. CNote: The passage states that 25% of the people born in seventeenth-century France die within a year of being born, and another 25% of those people die before the age of 20 (lines 9–12). The question stem tells us that this resident of seventeenth-century France began having children at age 20, and died at age 40. In those 20 years, then, 25% of the children would die within a year, and another 25% of the original number would die by the age of 20. Therefore, 50% of the total would die by the time the parent died at age 40.A: This number is too low. Over 10% (25%) would probably have died in their first year of life, and another 25% would be

likely to die before the parent died 20 years later.B: This number is too low. 25% would probably have died in their first year of life, and another 25% would be likely to die

before the parent died 20 years later.C: Yes. Half of the children would be expected to die by the end of those 20 years.D: This number is too high. 75% would be expected to die by age 45, that is, 25 years after the parent had died.

118. AA: Yes. By saying that “death had no more relation to old age than to any other period of life” (lines 6–8), the author

indicates that people commonly died at a relatively young age of causes that had nothing to do with aging. Later in the passage, the author states that increases in average life expectancy were “due mainly to success in reducing infant, childhood, and maternal mortality” (lines 19–21). Therefore, we can conclude that, even though older people could expect to live even longer than before (lines 23–25), the increase in average life expectancy was due more to better health in early life (infants, children, and women in childbirth) than to improved health of the elderly.

B: Be careful not to speculate beyond the scope of the passage. Just because infants and children were more likely to die in the past does not necessarily mean that parents cared less about those infants.

C: As in choice B, you need to be careful to stay as close as possible to the information provided in the passage. While it may make sense to say that if a person believes they are more likely to survive into adulthood and beyond they might (or should) be more concerned with long-term planning, the author never raises this issue in the passage.

D: As in choices B and C, this statement has no support in the passage. The author draws no connection between how many elderly people there are on one hand, and the respect accorded those people on the other.

119. DA: Both the passage and the new information in the question stem describe the attitude of people in retirement communities

in positive terms (lines 35–38 and 68–73). The wording of this choice (“ambivalence”) indicates that the retirees are questioning or doubting their decision. Finally, the passage does not describe moving into a retirement community in terms of “breaking with the past.”

Page 25: AAMC 7R Solutions

25Verbal Reasoning Solutions

B: While the positive tone (“satisfaction”) is appropriate, the credited response must also match up with some statement or theme in the passage. The author of the passage does not suggest that moving into a retirement community constitutes becoming independent of oneʼs family, or that independence is a major concern for retirees.

C: We have no way of inferring that the new arrivals were leaving difficult situations behind. Be careful not to read too much into the directorʼs use of the word “pilgrims.” While the pilgrims who came to North America from England may have been leaving behind religious persecution and economic hardship, there is nothing in the passage to suggest that people moving into retirement communities are similar or analogous in this respect.

D: Yes. The author of the passage tells us that “many gerontologists concluded that the elderly found in their segregated lives the advantages…advertised in the real-estate brochures” (lines 70–73). The author also states that “many retirees welcomed these complexes as a new adventure” (lines 37–38), and that many such developments have “evolved…into communities with traditions of their own” (lines 60–62). We can infer that having common traditions would likely involve having similar values or goals, and the word “optimism” in the answer choice is appropriate to the positive attitude expressed by the director. It also matches the positive attitude of retirees towards their “new life” described by the author of the passage.

120. CA: While this relationship is suggested by the passage, it has no direct relevance to the comparison between seventeenth-

century France and the twentieth-century United States.B: Nowhere in the passage does the author suggest that people believe demographic changes are unique to U.S. culture.

Even if this were the case, the comparison between France and the U.S. would not counter that belief, as the author never mentions demographic change in France. The statistics about greater life expectancy are all from the U.S. (lines 16–30).

C: Yes. The author cites mortality statistics from seventeenth-century France as an example illustrating how, in the past, most people died before they reached old age. The author then goes on to discuss the “demographic of mortality” in the U.S. in 1900 and in 1980 to illustrate how average life expectancy has significantly increased due to medical advances, and that many more people now live long enough to be considered “elderly.” Therefore, the discussion of France serves to introduce the authorʼs contrast between average life expectancy in the past and today, and to support the claim that “for an entire generation to reach old age with its membership almost intact is new” (lines 4–6).

D: The passage presents a sharp contrast between mortality in seventeenth-century France and the U. S. in 1900 on one hand, and the U.S. in 1980 on the other. The author states, “In the last hundred years, the demographics of mortality have changed more than in the six previous centuries” (lines 16–18); we are not told that longevity continues to increase in an “ongoing trend.” Furthermore, the intent of the reference to France is to show how people used to commonly die as infants, children, or young adults. The author distinguishes this issue from that of “longevity” (lines 21–25), which is the likelihood that a person in the later stages of adulthood (e.g., age 60) will live even longer.

121. DA: The fact that there are more elderly people does not explain why the life expectancy of an infant has increased by more

than the additional years a person at age sixty can expect to live. The difference is explained in the passage by the fact that most of the improvement came though reduced mortality of children and women in childbirth (lines 19–21). A person of sixty has already benefited from those improvements, and so can expect less of a payoff over the rest of his life span.

B: The passage compares the longevity of a white male of sixty in 1900 and in 1978. These numbers tell us nothing about what happened after 1978.

C: There is no evidence in these statistics or in the rest of the passage that women live longer than men. The statistic dealing with longevity is specific to men, and the comparison drawn is between overall life expectancy (at birth) and increased longevity of men who have already attained a certain age.

D: Yes. The author provides these statistics in part to contrast the 26.3-year (from 47.3 to 73.6 years) increase in average life expectancy of a newborn (due largely to “success in reducing infant, childhood, and maternal mortality”) on one hand with the 2.8-year increase in the additional years a white male could expect to live after the age of sixty (lines 23–25) on the other hand. The author labels the former “average life expectancy” and the latter “longevity.” People are not living that much longer overall (longevity), but more people are surviving past the age of sixty-five (overall life expectancy).

Page 26: AAMC 7R Solutions

MCAT Practice Test 7 Solutions26

Passage VII (Questions 122–127)

122. CA: The authorʼs critique of the self-serving motivation of those who “sell” such theories (lines 30-34) and of the long-term

counterproductive effect of their implementation (lines 75–77) is enough to support the label “unethical.” However, the author does not believe they are practical or effective; they are less than ethical in part because they are ineffective or impractical. For example, the author calls them a “confused approach” (line 70) and states, “In the long run, however, it is the forgotten parents and the children themselves who will pay for the short-sighted ambition of this policy” (lines 75–77).

B: The author says nothing good about these theories and would be more likely to agree that they are unethical than ethical. Furthermore, the author states that such theories or approaches represent “the usual practice at the professional level of education of treating learning as an abstraction that has little to do with the learner” (lines 34–37). Therefore, such a theory would likely be generally accepted.

C: Yes. See the explanations for choices A and B. The author thinks that such a theory or “confused approach” (lines 69–70) would be ineffective, because it is not based on actual experience with and knowledge of how children learn (lines 48–54). The author also bemoans the fact that despite this flaw, such theories are “the usual practice at the professional level of education” (lines 34–35).

D: See the solution for choice C. While the author would agree that it is impractical, the passage argues that such theories are ineffective.

123. DA: This choice is too extreme. The author doesnʼt go so far as to claim that these promoters are out to control the entire

educational system. Furthermore, the passage never mentions “consumers” as a target or focus.B: This choice is too extreme. The author does believe that these promoters are selling a theoretical approach that is conceptually

and practically flawed. The author also states that they are doing so in part “to keep their business financially solvent” (lines 31–32). However, the author does not go so far as to call this fraud, or to claim that the promoters are making a great deal of money.

C: The author refers to the promoters ̓own business (lines 30–32) but never indicates that corporate sponsorship is involved.D: In paragraph 7, the author states that “the advocates of generalized readiness are clearly directing their appeal to

school administrators” (lines 55–57). The author goes on to state that the promoters suggest that, “An administrator wise enough to adopt the readiness program is promised higher percentages on standardized tests and more content teachers” (lines 63–65).

124. CA: This choice is only partially supported by the passage. It is true that the author states that parents favor developmentally

appropriate instruction (lines 30–34 and 57–59). However, the fact that parents support it is not a reason why administrators reject it. While promoters of the readiness program may label parents who favor other approaches as “uncaring” (lines 57–59), the author does not describe administrators as sharing this view. Furthermore, the author argues that this approach does in fact reflect the views of those who know how children learn. For example, the author states that advocates of this approach such as Piaget and Gesell “had direct experience with children in an educational setting, and both contributed profoundly useful principles to the field of education” (lines 42–44).

B: The author suggests that administrators are reluctant to adopt the developmentally appropriate approach because they are more “concerned with uniformity” (line 75). That is, the readiness approach does not require as much attention to the specific needs of different students, and so is easier to implement. The author states, “Readiness, although a confused approach, is easily implemented because its promoters are positioned to move immediately” (lines 69–72). The author gives no indication that administrators believe that developmentally appropriate instruction is unproven or that it would require extensive research; their concerns relate more to convenience.

C: Yes. Through the passageʼs description of developmentally appropriate instruction, it is clear that it involves responding and adapting to the specific needs and abilities of children at different developmental stages (see lines 30–34 and 41–47). The author also states that administrators are reluctant to implement it because they are more concerned with “uniformity” (lines 72–75), and because the readiness approach as an alternative is “easily implemented because its promoters are positioned to move immediately” (lines 69–72). Lastly, the author tells us earlier in the passage that “the usual practice at the professional level of education” is to treat “learning as an abstraction” (lines 34–37). A developmentally appropriate approach, which approaches learning through practical

Page 27: AAMC 7R Solutions

27Verbal Reasoning Solutions

observation and direct experience (lines 37–47 and 48–55) would then be a departure from traditional practices, and so would be less convenient than the readiness program.

D: It is the readiness program, not developmentally appropriate instruction, that is “slickly presented” (line 69). Furthermore, the author does not discuss cost effectiveness of either program.

125. BNote: The correct answer will compare something that is more uniform and generic like the readiness program to something that is more individualized, like developmentally appropriate teaching. The two scenarios also need to be presented in that particular order, in order to match the relationship presented in the question stem.A: The passage does not indicate that developmentally appropriate teaching is more technologically advanced than School

Readiness.B: Yes. A yearly form letter is generic, not taking the specific characteristics of each recipient into account (like School

Readiness). Personal notes would be adapted to each individual (like developmentally appropriate teaching).C: This choice reverses the comparison. Casting a line with a fishing rod would be more specific and individualized than

dragging a net behind a boat, which entails pulling in all the fish in the vicinity without focusing on and approaching each one as an individual.

D: This choice mentions a variety of dishes prepared in a restaurant, while the author suggests that School Readiness does not provide a variety of approaches appropriate to different developmental levels. Furthermore, while a family dinner at home may be more personal, it is not necessarily adapted or adaptable to the needs of each individual. This is more likely to be possible in a restaurant, where a greater variety of foods is likely to be available.

126. AA: Yes. The author states, “The appropriate instrument for measuring educational effectiveness is a test noted for its

reliability and validity” (lines 11–13). Therefore, we can infer that this test of high-school students could measure how effective their education was in preparing them for college, that is, to evaluate the usefulness of high-school curricula.

B: There is no information in the question stem or in the passage that tells us how a test of the effectiveness of a studentʼs education would also tell us something about how fairly or honestly the students were graded.

C: The scores themselves could not be used to evaluate the reliability or appropriateness of the test that generated those scores. The test itself and its underlying principles would need to be evaluated by some standard external to the test itself in order to “[establish] its credentials” (line 15–18).

D: Compare this choice to choice A. It is reasonable to draw a connection between high-school test scores, the students ̓performance in college, and the value or effectiveness of the high-school education. However, it is much more of a stretch to say that these scores could be used to predict a studentʼs future income. Income depends on much more than just oneʼs high-school education.

127. CA: Neither the passage nor the question stem leads us to conclude that the author would believe that promoting language skills

at this age is unreasonable, or developmentally inappropriate.B: Given the authorʼs unrelentingly critical view of the Readiness Program in the passage (see lines 28–40, 44–47, 48–54,

and 69–77), this would be too easy and rapid a concession to the view that the program may have some value. To make this concession, the author would need the kind of evidence referred to in paragraph 2, not just a newspaper report.

C: Yes. Given the authorʼs deep suspicion of the Readiness Program (see the explanation for choice B), we can infer that the author of the passage would doubt the validity of a newspaper report showing the programʼs positive effects.

D: This choice is too extreme. While the author would likely remain deeply suspicious of the report and of the program, apparent or reported success in the teaching of language skills would not itself be seen by the author as irrelevant to the goals of education. A finding can be relevant and yet at the same time inaccurate.

Page 28: AAMC 7R Solutions

MCAT Practice Test 7 Solutions28

Passage VIII (Questions 128–132)

128. CA: The authorʼs response is directly relevant (see the solution for choice C). This choice itself incorrectly suggests that the

author assumes the objector does imply that people should be forced to disobey their consciences. The issue of being forced to disobey oneʼs conscience is not directly involved in either the objection or in the authorʼs response. The authorʼs reply that “the designated decision makers could decide anything, too” is meant to show why we should “let people decide for themselves what is right and wrong” (lines 45–48) since members of a government are not inherently better, or more moral, decision makers.

B: The authorʼs response is relevant (see the solution for choice C). While the author does make this statement in the passage (lines 3–4), it does not invalidate or make irrelevant the authorʼs response to the objection. The author argues that while the burden may be heavy, an individual is still obliged to carry it for him or herself (lines 6–9).

C: Yes. The authorʼs response is a logically valid refutation of the objection. The author argues in the passage that “A government is made up of individuals who are fundamentally similar to me, and ʻto err is human ̓applies to us all” (lines 53–55). Since the objection does not indicate that politicians are in fact better decision makers than the average person, the authorʼs response that what applies to us applies to politicians (they could decide anything) is directly relevant to the objection.

D: The point of the author is that we are responsible for our own decisions and that individuals should make those decisions for themselves. The implication that politicians should also obey their consciences is not directly relevant to the argument about whether or not it is morally valid to hand over moral authority for our own decisions to those politicians. The author argues, “Since someone has to decide what is the right thing for me to do, it may as well be me” (lines 50–52). The issue is the transferability of responsibility from an individual to the government, not the moral responsibility of the government for its own actions.

129. CA: This choice weakens rather than supports the passage. The author argues that even if one has “made a promise to obey, one

is still in fact free to obey or disobey, to keep or break that promise” (lines 14–15). If someone is incapable of breaking a promise, it would weaken the authorʼs claim that it is impossible to enslave “your will to someone elseʼs will” (line 12). Once the promise is made, if the person has no option to break it, they would no longer be morally responsible.

B: The author writes that “the responsible individual will often have to take into account what other individuals are doing when that individual is calculating the consequences of his or her action” (lines 33–36). However, this is a different issue from that of the possibility of putting oneʼs will under the control of another by promising to obey that person. The fact that the people described in the question stem may not make decisions based in part on the effects of their actions on others does not affect the authorʼs argument that they should do so.

C: Yes. This choice weakens the passage. The author argues that even if one has “made a promise to obey, one is still in fact free to obey or disobey, to keep or break that promise” (lines 14–15). If someone is incapable of breaking a promise, it would challenge the authorʼs claim that it is impossible to avoid responsibility for oneʼs own actions by transferring that responsibility to someone else (lines 20–23). Once the promise is made, if the person has no option to break it, they would no longer be morally responsible.

D: See the explanation for choice B. The new information in the question stem is not directly relevant to the authorʼs suggestion that other people matter. Therefore, it neither supports nor challenges the passage.

130. ANote: The theme of the passage is stated in the first sentence: “In a fundamental sense, every person is morally responsible for deciding what is right and what is wrong and acting accordingly.”A: Yes. In paragraph 4, the author states that “if the government requires a moral person to act in a particular fashion

and that person complies, it is because the person believes it to be the right thing to do, not simply because the government requires it” (lines 24–28). This is one application of the overall theme of the passage. Therefore, it would be consistent to say that laws created by the government would be legitimate only if they were consistent with the will of the governed. A law that was not in accordance with the will of the governed would illegitimately punish moral people for doing what they thought was right, that is, for acting in accordance with their own moral responsibility.

B: This choice contradicts the main theme of the passage. The author believes that people must in the end decide moral issues for themselves: “the final arbiter is the individual” (line 30).

Page 29: AAMC 7R Solutions

29Verbal Reasoning Solutions

C: The author does say that we must take effects on others into account, and that maintaining order (for example by driving on the same side of the street as everyone else [lines 33–39]) may in fact be beneficial. However, the promotion and advantages of order are not by themselves sufficient reasons to obey a law. Rather, when one does obey, it should be because one has decided that the action is moral in and of itself. Note that the author never suggests that laws should be obeyed in all or most cases. The author states, “That the government does require a particular behavior may be an important factor to be taken into account, but the final arbiter is the individual” (lines 28–30).

D: The author does recommend “if you are ignorant, consult an expert,” but then goes on to say, “but when to consult, whom to consult, and how to decide between conflicting experts is still the individualʼs problem. And so is the final decision” (lines 60–63). The author makes no distinction between individuals in general and lawmakers in particular (“ ʻto err is human ̓applies to us all” [lines 54–55]). Therefore, the author would say that lawmakers should ultimately rely on their own judgement, even if it is fallible.

131. CA: “Conflicting experts” is not at issue here, but rather a potential conflict between the decision of a government official and

of a private citizen, with the official being much more likely to be correct.B: See the solution for choice C. The new information in the question would undermine, not support this assertion. The

author supports the argument that individuals must ultimately decide for themselves what is right with the claim that “A government is made up of individuals who are fundamentally similar to me, and ʻto err is human ̓applies to us all” (lines 53–55). If officials were ten thousand times more likely to be correct, it would suggest that perhaps sometimes we should just do as commanded, without using our (much more fallible) individual judgement.

C: Yes. See the explanation for choice B. The author argues that government officials and private individuals are essentially equivalent in their capacity to make moral judgements (lines 48–55). If officials were so much more likely to be correct, it would weaken the authorʼs claim that “since someone has to decide,” it might as well be the individual (lines 50–52). Note also that the author writes, “Sometimes a government member may claim to have access to more information or expertise in a particular matter, in which case I will listen to that person and may very well go along with him or her because I believe that to be a strategy more likely to achieve a satisfactory result” (lines 55–60). If officials were ten thousand times more likely to be correct, then one might say the final arbiter might as well be an official.

D: If government officials were much more likely than private individuals to make correct moral decisions, it would have no relevance to or impact on the authorʼs argument that moral responsibility can be a heavy burden (lines 3–5).

132. BNote: The central thesis of the passage is stated in the first sentence: “In a fundamental sense, every person is morally responsible for deciding what is right and what is wrong and acting accordingly.” The rest of the passage supports and expands upon this theme.A: While this choice is consistent with the passage (lines 33–39), it does not in and of itself support the central thesis about

individual moral responsibility.B: Yes. See also the explanation for choice D. This new information simply indicates that in some cases people find it

beneficial to follow the law. It tells us nothing about how individual moral responsibility relates to the law or to a decision to follow the law. Therefore, it neither strengthens nor undermines the central thesis.

C: See the solution for choice B. The fact that in some cases people find it advantageous to follow the law is not inconsistent with the authorʼs overall argument that people must ultimately decide issues of right and wrong for themselves.

D: See the solutions for choices B and C. The author states that, “The fact that the government has decreed ʻEveryone shall drive on the left ̓may result in people so arranging themselves, and conformity with the traffic flow is a desirable thing” (lines 36–39). Therefore, the information in the question stem is entirely consistent with the passage and does not indicate that the central thesis must be changed or modified.

Page 30: AAMC 7R Solutions

MCAT Practice Test 7 Solutions30

Passage IX (Questions 133–137)

133. BA: Gibbon sees human nature as defined by “humanityʼs predilection for factionalism” (lines 3–4) and “the tendency towards

strife” (line 64). The description of Rome in this answer choice presents a positive picture, one of patriotic virtue and unification despite Romeʼs multi-ethnic character. Therefore, this particular assertion does not illustrate or follow from the negative attributes of human nature discussed by Gibbon.

B: Yes. Gibbon defines the role of a good government as a mediator of disputes (lines 69–71) because he views human nature as defined by factionalism (lines 2–4) and strife (lines 64–65).

C: This assertion does not follow from Gibbonʼs view that human nature tends towards factionalism and strife (lines 2–4 and 64–65). Although human nature is one of these forces of history, this answer choice does not present a view that would follow from Gibbonʼs beliefs about human nature.

D: Compare this choice to choice B. While tyranny could in some cases be related to the human tendency towards factionalism and strife, it does not make sense to say that Gibbon claimed that a Roman emperor was the first tyrant because humans are divisive and contentious.

134. BA: Gibbon sees history as essentially defined by conflict and factionalism (lines 3–4 and 64–65). The widely supported civil

rights movement in the U.S. would weaken, not strengthen Gibbonʼs argument by showing that people may be able to overcome their tendency to separate themselves into factions and to pit those factions against each other.

B: Yes. Gibbon sees history as determined by “humanityʼs predilection for factionalism” (lines 3–4). The author provides evidence for this view in part by describing in paragraph 4 some modern instances of factionalist strife that parallel events in Roman history. Ethnic conflict in Bosnia would be analogous to the examples of the Crimean Chersonites and the Caucasus (lines 41–46) given in the passage.

C: While the invasion of Kuwait by Iraq would fit with Gibbonʼs view of human nature as defined by a “tendency toward strife” (line 64), this answer choice focuses on the U.N. response to that invasion. An attack meant to stop an invasion could be seen as an attempt to limit strife.

D: As in choices A and C, this choice describes movement away from factionalism and/or strife, rather than an example of how “humanityʼs predilection for factionalism” (lines 3–4) and “tendency toward strife” (line 64) drive history.

135. DA: Rome is described as a polity “founded on patriotic virtue” (line 35), and yet Rome also knew “violent mobs screaming

noble platitudes in order to remove a tyrannical ruler, only to see another take his place” (lines 62–64). Therefore, it is unlikely that Gibbon would attribute domestic peace to patriotism.

B: Nothing in the passage would lead us to conclude that having a vast territory contributes to peace.C: Be careful not to bring outside knowledge or opinion to bear. Nothing in the passage suggests that a country will unify (and

stay unified over a long period of time) in the face of an external threat.D: Yes. This is the only choice of the four that can be connected to Gibbonʼs views as discussed in the passage. Gibbon

located “humankindʼs best hope in moderate politics and flexible institutions that would not become overbearing” (lines 56–58) and believed that “a state or an empire can endure only if it generally limits itself to adjudicating disputes among its citizens”(lines 69–71). The U.S. Constitution is the only factor listed in these answer choices that could relate to political institutions and their role in limiting conflict and thus in maintaining stability.

136. DA: This choice is not specific enough to Gibbon or to his views nor does it provide sufficient evidence upon which to conclude

that such a bias would invalidate his conclusions.B: This choice is consistent with the passage. Gibbon himself describes the influence upon history of certain leaders, for

example, “the story of the Empireʼs restoration in the third century under the able rule of Claudius, Aurelian, Probus, and Diocletian” (lines 12–15). While the author says that in the long run “only patterns, rather than individuals, endure” (lines 20–21), the author does not go so far as to suggest that Gibbon believes that leaders have no influence whatsoever on world history.

C: This would be consistent with the passage. The Roman Empire itself lasted for hundreds of years. The author writes about “the sheer accumulation and repetition of events over centuries” (lines 15–16) as part of the story of the Empireʼs restoration. The passage also mentions how Milan and Nicomedia functioned as “capital cities decades before the formal

Page 31: AAMC 7R Solutions

31Verbal Reasoning Solutions

division of the Empire into western and eastern halves and almost two centuries before Rome officially ceased to be the imperial capital” (lines 26–29).

D: Yes. The author describes “Gibbonʼs certainty that the tendency toward strife is a consequence of the human condition—of its variety of racial, cultural, and economic experiences, which no belief system…can overcome” (lines 64–67). The author also cites Gibbonʼs belief that “human nature never changes and that humanityʼs predilection for factionalism…is the determinant of history” (lines 2–5). If it is common to have societies that are both ethnically diverse (and so that Gibbon would expect to break apart into factions) and stable, it would challenge or undermine Gibbonʼs presumed view of history.

137. BA: This choice is too strong to be supported by the passage. The author states that Gibbon and Madison were influenced by

“Enlightenment rationalism” (line 54). While the Enlightenment did not wipe out all corruption (lines 47–52), that does not mean that it had no influence. Even if the statement in this choice were supported by the passage, it would be evidence of lack of change, not of gradual change.

B: Yes. In paragraph 3, the author lists the case of these two cities as an example of “insidious transformations” (line 23) or gradual changes. Milan and Nicomedia acted as capital cities for hundreds of years before Rome ceased to be the imperial capital and was replaced.

C: The author does not suggest that this alliance came about gradually (lines 41–46). Furthermore, it comes in the context of the authorʼs discussion of patterns of history, not of change.

D: The lack of distinction between members of the Roman middle class (lines 33–37) is not described as a “transformation” or change and is not portrayed as something that came about slowly or gradually.

Page 32: AAMC 7R Solutions

MCAT Practice Test 7 Solutions32

BIOLOGICAL SCIENCES SOLUTIONS

Passage I (Questions 138–142)

138. A. Theory II states that metabolic rate and free radical production are directly proportional. The question text states that, of the three species listed, humans have the lowest metabolic rate, rats have an intermediate metabolic rate, and mice have the highest metabolic rate. Thus humans should have the lowest rate of free-radical production (and subsequent urinary elimination), rats an intermediate rate, and mice the highest rate. The only graph that depicts this is choice A.

139. D. Theory I states that cells are preprogrammed to undergo 50 doublings. Theory II states that life span is related to free radical production, and that vitamin E can increase life span by eliminating free radicals. Since the cells in culture had already doubled 30 times, an additional 20 doublings would support Theory I, and more than 20 doublings would support Theory II. The questions asks for support of Theory II, so choice D is correct and choice C is wrong. Note that choices A and B support neither theory, since they indicate a reduced life span.

140. A. Theory I states that aging is triggered by hormones; thus the removal of an endocrine gland and subsequent increase in life span supports this theory. The evidence does not support Theory II, which states that dietary restriction increases life span, because in this case the octopus increases eating and lives longer (choice C is wrong). Theory II says nothing about genetics (choice B is wrong), and all living organisms undergo aging (choice D is wrong).

141. D. The passage states that vitamins E and C eliminate free radicals, thus they cannot contribute to aging (choice D does not lead to aging and is the correct answer choice). Absorption of ultraviolet radiation leads to free radical production (choice A could lead to aging and can be eliminated). Partially reduced oxygen species are themselves free radicals (choice B could lead to aging and can be eliminated). Metabolic neutralization of toxins leads to free-radical production (choice C could lead to aging and can be eliminated). Note that even if choices A, B, and C were not known to lead to free-radical production, choice D is a clear contradiction of the information in the passage.

142. C. The evidence for Theory II states that dietary restriction increases life span of rats from 125 to 185 weeks. This is shown in choice C, where the surviving fed rats drop to zero at approximately 125 weeks, and the surviving fasting rats drop to zero at approximately 180 weeks.

Passage II (Questions 143–148)

143. A. The sympathetic nervous system (“fight or flight” system) causes the heart rate to increase. Since diving is a new and possibly exciting experience for Sarah, it seems likely that her sympathetic nervous system would be activated. The parasympathetic nervous system leads to a decrease in heart rate (choice B is wrong). Although hypoxia would lead to an increase in heart rate, at sea level there is plenty of oxygen in the atmosphere to fully saturate the hemoglobin so hypoxia is unlikely (choice C is wrong). Elevated core body temperature would also cause an increase in heart rate, however this elevated rate would then be seen each time she dove. The passage states that during the first two diving events Sarahʼs heart rate was elevated, but that by the third event it was no longer elevated. This is consistent with a new exciting event becoming less stressful with experience (choice D is wrong).

144. C. The passage states that the higher heart and breathing rates were consistently elevated during the skiing trip. The most likely cause of this would be hypoxia due to the higher altitude; the breathing rate increases to bring in oxygen more quickly, and the heart rate increases to deliver that oxygen to the body more effectively. Activation of the sympathetic nervous system due to the stress of the new experience would increase heart and breathing rate, but as Sarah gained experience, skiing would become less stressful and this could not account for a prolonged increase in heart rate (choice A is wrong). The parasympathetic nervous system and hypothermia would cause decreases in heart and breathing rate (choices B and D are wrong).

Page 33: AAMC 7R Solutions

33Biological Sciences Solutions

145. A. Since urine production is directly related to blood pressure, increased blood pressure would certainly lead to increased urine formation. Excitement or anxiety would activate the sympathetic nervous system, which would increase blood pressure, and this would lead to the increased during production (choice A is correct and choice B is wrong). It is unlikely that water would be absorbed from the ocean; the skin is a fairly watertight barrier, and further, if anything, water would be drawn from the skin due to the higher osmotic pressure of the ocean (choice C is wrong). While it is true that evaporative water loss would not be possible while diving, ocean temperatures (even warm Caribbean Sea ocean temperatures) are typically cooler than body temperature, so the skin would not need to be cooled by evaporation.

146. D. Myoglobin is a protein found only in muscle tissue, so if myoglobin is present in the urine, muscle injury is likely (Item II is true). Note that only choice D contains Item II, so it must be the correct answer choice, and Item III must also be true. Since myoglobin is a relatively large protein, and the filtration apparatus of the kidney excludes proteins, it would normally exclude myoglobin from the urine. The fact that it is not, means that the kidney must have been damaged as well (Item III is true). Bone does not contain myoglobin, so a bone injury would not lead to myoglobin in the urine (Item I is false).

147. D. The brain stem regulates basic vital functions such as breath rate and heart rate, the cerebellum coordinates and smoothes body movement, and the hypothalamus regulates appetite (among other things). Only choice D has these listed in the proper order. Note that the cerebrum is involved in sending commands to muscles to contract, but the cerebellum coordinates their activity.

148. B. If the skin blood vessels were to remain constricted for great lengths of time, skin cells would not receive sufficient oxygen to survive. Skin tone (color) is the result of oxygenation, not the cause (choice A is wrong). While dilation of the vessels would lead to a decrease in blood pressure, it is unlikely that blood pressure would be excessively high because of other compensatory mechanisms that kick in when skin vessels must be constricted to conserve heat (choice C is wrong). While it is true that maintenance of muscle tone requires oxygen, dilating skin vessels would not supply oxygen to the muscles (choice D is wrong).

Passage III (Questions 149–153)

149. C. According to the Cahn–Ingold–Prelog priority rules, the highest priority goes to the group in which the atom attached to the chiral carbon has the highest atomic number. N has a higher atomic number than both C and H.

150. A. The phosphorus ylide is the best choice, by analogy to Step 4 in Figure 2. Compound 6 reacts with a similar reagent to yield Compound 7, containing a new C=C double bond. Both C and D can be eliminated because organometallic reagents (nucleophiles) will add to the carbonyl carbon to give an alcohol. Formaldehyde, choice B, can also be eliminated since it cannot react under neutral conditions with a ketone. An aldol reaction between these compounds would attach the new carbon group alpha to the ketone, not directly to the carbonyl carbon.

151. C. First count the number of stereocenters in the molecule: there are four. The maximum number of possible stereoisomers is 2n, where n is the number of stereocenters. Since n = 4, there are 24 = 16 possible stereoisomers.

152. C. The new bond forms between the atoms labeled b and c, so eliminate choices A and D. The Hc alpha proton can be removed to form an enolate, which nucleophilically attacks the beta carbon of the α,β-unsaturated ester (which is electrophilic due to conjugation to the ester carbonyl) in an intramolecular Michael reaction.

153. D. Due to the presence of the double bond, choices A and B can be eliminated. A CO2Et group contains one oxygen as part of the backbone chain linking two carbon groups, but the other is a carbonyl oxygen, making the functional group an ester, not an ether.

Page 34: AAMC 7R Solutions

MCAT Practice Test 7 Solutions34

Passage IV (Questions 154–160)

154. B. The passage states that reverse transcriptase is used to produce DNA from an RNA template. This is the reverse of RNA synthesis (transcription), in which RNA is produced from a DNA template. DNA replication produces new DNA from a DNA template (choice A is wrong), protein synthesis produces protein from an RNA template (choice C is wrong), and carbohydrate synthesis does not require the use of nucleic acids at all (choice D is wrong).

155. B. Antibodies are highly specific markers produced by B cells that bind to and inactivate foreign proteins (antigens). The great variability of the gp120 protein assures that there would be several different versions of this viral protein within a single host, and antibodies specific for one version may not be able to bind to and inactivate a different version. While it is true that antibodies are large, complex, and relatively uniform in structure, this would not account for their inability to neutralize all versions of the AIDS virus (choices A and C are wrong). Choice D is simply untrue; antibodies have two antigen-binding sites.

156. A. Individual nucleotides in DNA and RNA are connected by a phosphate molecule that bridges the 3ʹ hydroxyl group of the sugar on one nucleotide to the 5ʹ hydroxyl group of the sugar on another nucleotide. The sugar and nitrogenous base are attached at the 1ʹ carbon of the sugar (choice B is wrong), the phosphate and nitrogenous bases do not connect to one another (choice C is wrong), and the complementary bases between DNA strands are connected by hydrogen bonds (choice D is wrong).

157. D. Viruses do not carry ribosomes and rely on the host cellʼs ribosomes to carry out their protein synthesis.

158. C. If the virus has been “incorporated,” this means its RNA genome has already been reverse transcribed to DNA and inserted into the host cell (CD4 cell) genome. Thus its genetic information would be found in the nucleus, where the host cellʼs DNA is also found.

159. C. The CD4 receptor on the helper T cell is there for normal T-cell function; it is simply through chance that the virus was able to create a protein that could bind to the CD4 receptor and allow it access to the cell. Since the viral protein and the receptor bind to each other, this is not truly convergent evolution. Convergent evolution results in structures that have similar functions; for example, if the virus had developed a receptor similar to the CD4 receptor (rather than developing the ligand for the receptor) this would be an example of convergent evolution, however evolution of the ligand (the gp120 protein) is not convergent evolution (choice B is wrong). Coevolution benefiting the host and natural selection favoring mutation of the host would not have resulted in a receptor that would bind this devastating virus. Beneficial mutations for the host would have resulted in receptors that would not bind the virus (choices A and D are wrong).

160. D. B cells produce antibodies, so a virus that infects and destroys B cells would most severely affect antibody production. Foreign antigens are recognized by many different immune system cells, including T cells, B cells, phagocytes, etc. (choice A is wrong). Engulfing and lysing antigen is the job of macrophages and other phagocytes (choice B is wrong), and there is no specific cell involved in antibody binding. This is simply a protein–protein interaction (choice C is wrong).

Free-Standing Questions (161–164)

161. D. Since an alcohol can hydrogen bond and is more polar, it will have stronger intermolecular interactions with the silica gel. In thin layer chromatography, more polar compounds do not travel as far on the plate, and therefore have lower Rf values.

162. D. This question asks to compare the rates the SN1 and SN2 mechanisms under protic solvent conditions. Choices A and C can be eliminated since they are both secondary substrates, and will be slower than either a primary substrate if the SN2 mechanism predominates, or slower than a tertiary substrate if the SN1 mechanism predominates. Because a protic solvent is present, the SN1 mechanism must be favored. The more substituted substrate will react faster since it will yield a more stable carbocation intermediate.

163. C. The graph shows an increase in cyclin concentration during interphase and a decrease during mitosis. Thus, translation (synthesis) must be occurring during interphase, and proteolysis (degradation) must be occurring during mitosis (choice C is

Page 35: AAMC 7R Solutions

35Biological Sciences Solutions

correct and D is wrong). Simple replication of the gene or segregation of the chromosomes carrying the gene would not result in synthesis of the cyclin protein; in fact, protein synthesis is halted during replication and during mitosis (choices A and B are wrong).

164. B. The number of cells resulting from cell division is equal to 2n, where n is the number of cell-division cycles. Since the mouse cells divide every 10 hours, and 3 days is 72 hours, the cells would go through 7 cycles of division. Therefore, the number of cells produced would be 27 = 128 (which is between 50 and 500).

Passage V (Questions 165–169)

165. D. For C-5, the priorities of the groups are: 1) the ring oxygen because it has a higher atomic number than the C or H, 2) C-4, because it is more highly substituted than C-6, 3) C-6 because it has a higher atomic number than the H, and 4) H. Groups 1, 2, and 3 trace a clockwise arc with Group 4 behind the molecule, giving the stereocenter the R configuration. For C-7, the priorities of the groups are: 1) the OH group because of atomic number, 2) C-8 because it is more highly substituted than C-6, 3) C-6 because of atomic number, and 4) H. Groups 1, 2, and 3 trace a counterclockwise arc with group 4 behind the molecule, giving the stereocenter an S configuration.

166. B. Grignard reagents attack electrophilic carbonyl carbons to form new C–C bonds and yield alcohols. Choice A shows a hydride reduction of the ketone, choice C shows no reaction at all, and choice D shows a 1,4 addition of the methyl group and a hydride reduction of the ketone.

167. A. Grignard reagents are prepared from their corresponding alkyl halides and magnesium metal.

168. A. The only way for one structure to represent a mixture of isomers is if the molecule contains one or more chiral centers. The structure then represents both enantiomers, a type of stereoisomer, if the perspective of the bonds is not indicated (as in Compound 7). Only choice A shows a pair of enantiomers that corresponds to the structure of Compound 7 in the passage. Choices C and D are structural isomers.

169. D. Hydroxyl groups appear in the 3200–3600 cm–1 range of an IR spectrum, while carbonyl groups appear near 1700 cm–1.

Passage VI (Questions 170–174)

170. C. The colon (large intestine) has only two functions: to reabsorb water and to store feces. Digestion, absorption, and secretion of enzymes takes place in the small intestine (choices A, B, and D are wrong).

171. D. Antibodies are proteins with enormous variability. This is made possible by random B cell DNA rearrangement prior to transcription and translation of that cellʼs particular antibody. The result is an extremely diverse population of cells, each of which produces an antibody distinct from all others. Due to the randomness of the procedure, it is inevitable that some of the antibodies produced will recognize “self” antigens. Very early in development, any cell producing antibodies that recognize “self” are destroyed or suppressed (choice D is correct and C is wrong), so that autoimmunity does not occur (obviously, this is not a perfect system). Antibody–antigen interaction occurs extracellularly (choice A is wrong). The body does make antigens that the immune system can recognize, however the cells that recognize the self-antigens are suppressed (choice B is wrong). Consider organ transplantation; patients undergoing this procedure must take drugs to suppress their own immune system so that it does not attack the new transplanted organ—clearly bodies make antigens that can be recognized. We just usually do not detect our own.

172. B. The peritoneal cavity is the subdivision of the ventral body cavity that contains the abdominal organs, including the intestines. Penetration of the intestinal wall would allow the contents of the intestinal lumen (the inner hollow part of the intestines) to enter this cavity (choice B is correct and D is wrong). The perineum is the region between the external genitalia and the anus and is not a cavity at all (choice A is wrong). The pleural cavity is the subdivision of the ventral body cavity that contains the lungs (choice C is wrong).

Page 36: AAMC 7R Solutions

MCAT Practice Test 7 Solutions36

173. D. Remember that antigens are simply proteins, typically found on a cellʼs surface. Our immune system produces antibodies that bind to and mark antigens for destruction. Usually the antigens are foreign and are associated with pathogens, such as bacteria and viruses. Rarely, the antigens are our own, and are associated with our own cells. The antigen that is possibly involved in inflammatory bowel disease must be a surface protein on a cell in the digestive tract. DNA and mRNA are inside the cell, are not proteins, and could not lead to the inflammation described in the passage.

174. D. If the gene for inflammatory bowel disease were simply recessive, it would display clear Mendelian inheritance patterns. Incomplete penetrance and limited expressivity are departures from classic Mendelian inheritance (choices A and B are suggested by the lack of Mendelian inheritance and can be eliminated). If the disease were polygenic (caused by several genes), and the genes interacted in various ways, this could also cause the non-Mendelian inheritance pattern (choice C is suggested by the lack of Mendelian inheritance and can be eliminated).

Passage VII (Questions 175–180)

175. C. The fatty acid group was not given any supplementation to reduce oxidation of the fatty acids, and they had a reduced blood flow. The data are consistent with choice C. Because vitamin E is an antioxidant, the fatty acid + vitamin E group would have more unoxidized fatty acids than the fatty acid group, and therefore should not have such a reduced blood flow; in fact the data shows that their blood flow was increased (choice D is wrong). The data show that vitamin E alone increases blood flow (choice A is wrong). While it is true that vitamin E alone increases blood flow more than fatty acids alone, we are not asked to compare these two groups (choice B is true but does not answer the question).

176. A. Since norepinephrine is the neurotransmitter used by the sympathetic nervous system, a change in sympathetic activity should lead to a change in norepinephrine content in the blood. Acetylcholine is the neurotransmitter used by the parasympathetic system (choice B is wrong). Epinephrine can also stimulate the sympathetic system; a lack of epinephrine receptors in skin blood vessels means that the sympathetic nervous system could not be affecting these vessels (choice C is wrong). In vivo dilation of blood vessels would increase blood flow, not decrease it (choice D is wrong). Note also that dilation of blood vessels (except in rare instances) is not typically a function of the sympathetic nervous system.

177. D. Blood pressure is highest in the ventricles of the heart, and goes down as the distance from the heart increases. Of the choices given, the capillaries are farthest from the heart and would have the lowest pressure. Note also that pressure decreases as cross–sectional area increases, and that capillaries have the highest collective cross-sectional area of all the vessels in the body.

178. C. The researchers could not monitor the subjects 100% of the time during the study to make sure they were following their instructions. Therefore, they must assume that the subjects followed all procedures properly, and that no lifestyle changes were made during the study. This is to ensure consistency, so that any observed changes in blood flow would be known to be from the dietary supplements being tested, and not from some other change in habits. Fatty acids and vitamin E were the two substances being tested, so no assumption should be made regarding their effect on blood flow (choices A and B are wrong). Blood pressure was not one of the parameters measured in this study (choice D is wrong).

179. D. Unsaturated fats contain one or more C=C double bonds, and thus are “less saturated” with hydrogen atoms than saturated fats. Saturated fats, with no C=C double bonds, are “fully saturated” with hydrogen (choice B is wrong). The number of carbon atoms do not differ between saturated and unsaturated fats (choice A is wrong), and both types of fats have carbon–hydrogen bonds (they are in fact, hydrocarbons; choice C is wrong).

180. D. Skin temperature was stated to be 32°C in all subjects, the age range was stated to be 18–28, and the blood flow was measured at the same time of day to eliminate the possibility of interference by diurnal (night/day) patterns; therefore, choices A, B, and C were all controlled and can be eliminated. Only blood flow, which was the factor being measured, was not controlled by the researchers (choice D was not controlled and is the correct answer choice).

Page 37: AAMC 7R Solutions

37Biological Sciences Solutions

Passage VIII (Questions 181–185)

181. C. The passage indicates three ways to stabilize a carbocation: resonance, the inductive effect, and hyperconjugation. Resonance requires the delocalization of charge and/or electrons, and is not related to the degree of substitution. Eliminate choice A. More highly substituted carbocations are better stabilized by induction and hyperconjugation, making C the best choice. Steric interference (choice B) would destabilize a molecule, and choice D is unrelated.

182. B. The passage states that Figure 1 represents the 1H NMR spectrum of the major product. The two signals in the spectrum indicate the compound has only two types of Hʼs (the small blip at 0 ppm represents TMS, the internal standard). Only the tert-butyl alcohol has two types of protons, which integrate for 1 H (the OH group) and 9 H (the three equivalent methyl groups).

183. B. Treatment of an alcohol with a strong acid results in protonation of a lone pair of electrons on the oxygen atom. This converts a poor leaving group (OH–) into a good leaving group (H2O). Choice A describes multiple steps in the conversion of an alcohol to an alkene. The first step of the reaction must then be choice B. Choice C would occur if the alcohol were treated with a strong base, not acid. Choice D does not occur since OH– is a poor leaving group.

184. D. The passage states that the second most abundant product was sec-butyl alcohol, which is represented by the peak labeled with a 3 in the gas chromatograph trace. Since the numbers represent relative amounts of material, the percentage of this compound can be calculated as follows:

185. D. Since the reaction was quenched with water, all products will be alcohols, and contain the OH functionality. Alcohol stretches appear between 3200–3600 cm–1.

Free-Standing Questions (186–190)

186. D. Bacteria are able to reproduce independently of any other organism as long as they are provided with a nutrient source such as glucose. Viruses, on the other hand, are unable to reproduce without their host cells (bacteria, animal cells, plant cells, etc.). Both bacteria and viruses can undergo mutation (choice A is wrong), and both bacteria and viruses lack a nucleus and therefore lack a nuclear membrane (choice B is wrong). Both bacteria and viruses contain protein in their outermost covering; virus have a protein capsid.

187. D. The Golgi apparatus is a stack of flattened membranes, and the smooth ER is also composed of folded membranes. They look similar under the microscope. The nucleolus appears as a dark, rounded structure within the nucleus (choice A is wrong), the mitochondria have an outer membrane surrounding a clearly folded inner membrane (cristae, choice B is wrong), and the plasma membrane is a well-defined boundary surrounding all the other organelles and cytoplasm. It is unlikely that it would be confused with the Golgi apparatus (choice C is wrong).

188. C. Hydrolysis of a triglyceride yields three equivalents of fatty acid and one equivalent of glycerol. Since the question states 2 moles of the fatty acid have the R side chain while only 1 mole has the Rʹ side chain, the triglyceride must have a 2:1 R:Rʹ ratio.

189. C. Most fats are carboxylic acid esters. If lipases can catalyze their hydrolysis as well as the hydrolysis of carboxylic acid esters that are not fats, then this demonstrates that enzymes can interact with different substrate molecules as long as they have similar chemical linkages (choice C is correct and choices B and D are wrong). There is nothing in the question text that suggests that lipases are anything other than proteins (choice A is wrong).

190. C. Let us represent the allele for red coat color with the letter R and the allele for white coat color with the letter W. The roan-colored colt must be heterozygous and have the genotype RW, and its white-colored mother must have the genotype WW. Thus the colt inherited the W allele from its mother and must have inherited the R allele from its father. There are two possible genotypes that could donate the R allele: RR or RW. The phenotypes associated with these genotypes are red or roan, so the colt father could have been either of these colors.

Page 38: AAMC 7R Solutions

MCAT Practice Test 7 Solutions38

Passage IX (Questions 191–197)

191. B. Simple increased proliferation of cells is not enough to cause cancer; somehow the cells must mutate and lose control of their own division. Mutation is more likely, of course, due to the rapid proliferation and subsequent DNA replication! However, the mutation must occur in the cells that are proliferating (in this case the stomach cells, which are somatic; choice B is correct and A is wrong). If the crowded mucosal cells remain in interphase, they have effectively stopped dividing, in which case cancer would be less likely (choice D is wrong). Just because the immune system fails to recognize bacterial antigens does not mean that cancer is imminent. In fact, the passage states that these bacteria frequently evade immune detection and that more than 75% of infected individuals do not develop cancer (choice C is wrong).

192. A. The stomach is extremely acidic (pH around 1) and this environment often denatures proteins. Antibodies are generally effective against bacteria; this is an unusual case (choice B is wrong). There is nothing in the passage to suggest immune suppression; if this were the case, H. pylori infection would be accompanied with many other infections as well (choice C is wrong). Antibodies are secreted from B cells into the plasma and into interstitial fluid, both extracellular spaces (choice D is wrong).

193. B. The passage states that different genes are expressed in different strains of the bacterium. There is nothing to suggest that different strains attack different hosts, or exhibit different resistance patterns, or exist in different countries (choices A, C, and D are wrong).

194. C. The passage states that the cagA gene leads to inflammation, which involves blood vessel dilation (choice D is wrong), subsequent swelling, and the movement of white blood cells (leukocytes) into the area. There is no reason to assume disruption of enzymatic activity or host cell protein synthesis. These are most often associated with viral (not bacterial) infection (choices A and B are wrong).

195. D. The bacterial genes are not incorporated into host chromosomes; this is a feature associated with viral infection (choice A is wrong). The passage states that most people do not eradicate the infection (choice C is wrong). Therefore, infected individuals who do not develop cancer must either have “robust” immune systems or must simply tolerate the infection without developing cancer. Since the definition of “robust immune system” is vague at best, and since no reference is made to this in the passage, tolerance to the infection is more likely (choice D is better than choice B).

196. A. A therapy for any kind of disease should be targeted against the cause of the problem. In this case the cause of the problem is the aberrant stomach cells that are dividing out of control and producing tumors, so the gene therapy should be directed there. Targeting H. pylori might help root out the cells that led to the out-of-control division (thus reducing the likelihood of future cancers), however not all stomach cancers are caused by H. pylori, and this would not help eliminate the existing cancer cells (choice D is wrong). Targeting the antibody-producing cells of the immune system would destroy the bodyʼs own natural defense against cancer (choice B is wrong), and there is no reason to target all body cells if only the stomach cells have cancer (choice C is wrong).

197. D. The primary function of the stomach is to grind and store food so that it can enter the small intestine slowly. Very limited digestion and absorption occur in the stomach. Acid is secreted to neutralize harmful pathogens and to initiate general food hydrolysis, and pepsinogen is secreted to initiate protein digestion. All other macromolecules are digested in the small intestine by pancreatic and intestinal enzymes (choices A, B, and C are wrong).

Passage X (Questions 198–204)

198. A. In an intact embryo, the AB cell produces more kinds of tissue than when it is isolated. Clearly, something about being with the other cells in the embryo aids in its differentiation. The fact that P1 cells develop identically regardless of whether they are intact or isolated supports the hypothesis that cell-to-cell communication is not needed for determination of fate (choice B is wrong). Choices C and D do not discuss results of Experiment 1.

Page 39: AAMC 7R Solutions

39Biological Sciences Solutions

199. B. Since the P1 cells were able to divide and differentiate normally in the absence of the AB cells, but the AB cells were not able to differentiate normally, this indicates that the P1 cells must be necessary for normal AB cell differentiation (choice B is correct and choice A is wrong). Since the P1 and P2 cells were not separated, interactions between them cannot be determined (choice C is wrong), and the zygote divides to form AB and P1 cells; the zygote and the AB cells are not found together and cannot influence one another (choice D is wrong).

200. D. In Experiment 2, preventing translation (protein synthesis) prevented normal development, whereas preventing transcription (mRNA or rRNA synthesis) had no effect on the cells. This indicates that the signal must be a protein (choice D is correct and choices B and C are wrong). Note that this is the most likely choice anyway, as proteins are almost always the signaling molecules between cells. Nothing was done in Experiment 2 to prevent DNA replication, and in any case, DNA is never used as a signaling molecule (choice A is wrong). 201. A. Gut differentiation only occurred when EMS cells were present (choices C and D are wrong) and occurred immediately in the P2–EMS cell combination (choice A is correct and B is wrong).

202. B. The gut differentiation substances must be segregated until gut differentiation occurs. Use Figure 1 and work backwards from “gut” to see the pathway: zygote to P1 to EMS to E. Not other pathways leads to the gut.

203. C. From Figure 1 it can be seen that gut only arises from blastomere E. Neuronal tissue arises from both ABs, from MS, and from C (choice A is wrong), and muscle arises from both ABs, from MS, from C and from D (choice B is wrong). Germ cells are not somatic (choice D is wrong).

204. C. Cells did not develop normally in the absence of other cells, so cell-to-cell signaling was necessary. The only answer choice that states this is choice C. Normal development does not involve cell separation (choice B is wrong) and none of the experiments tested separation of the cells into the three primary tissue layers (choice D is wrong). Choice A is tempting because Experiment 2 showed that an inhibitor of transcription did not disrupt normal development. However, an inhibitor of translation did disrupt development, and transcription is necessary for translation to occur, thus normal nematode development must involve transcription at some level (choice A is wrong).

Passage XI (Questions 205–208)

205. B. Use the arrow pushing mechanism given in Equation 2 of the passage. The allyl vinyl ether reacts to form an intermediate that has a carbonyl group and a new C=C double bond at opposite ends of the carbon chain. Choice A is missing the C=C double bond, choice C has the oxygen embedded in the carbon chain instead of in a carbonyl functional group, and choice D puts the carbonyl in the middle of the chain instead of in the terminal aldehyde position.

206. A. If the entropy of activation is less than zero, the entropy of the system is decreasing and the system is becoming more ordered. Eliminate choices B and D. Chemist 1ʼs mechanism requires two parts of the molecule to be brought together to form a cyclic transition state which is more ordered than the starting material. However, Chemist 2ʼs mechanism requires a bond to break, making two intermediates from one starting material. This entropy change is positive (the system becomes more disordered).

207. B. No cross products can form if the mechanism of the reaction is concerted (Chemist 1) since one bond is broken at one end of the molecule at the same time a new bond forms at the other end. There is no chance the side chain can leave the starting material. However, if the first step of the reaction involves bond cleavage to form two intermediates (Chemist 2), the phenolate ion of one starting material could react with the carbocation of the second starting material as they diffuse through the reaction flask.

208. D. Alcohol stretches appear between 3200 and 3600 cm–1.

Page 40: AAMC 7R Solutions

MCAT Practice Test 7 Solutions40

Free-Standing Questions (209–214)

209. B. The fact that the children display the disease and the parents do not means that the disease is caused by a recessive allele (choice A is wrong). The fact that more men than women have the disease (in fact only men have the disease) indicates that it is most likely sex-linked (choice C is wrong). Because only sons inherited the allele, and sons get their Y chromosome from their father, and neither father has the disease, the disease must be linked to the X chromosome and have come from the mother (choice B is correct and D is wrong).

210. D. The gallbladder stores bile made in the liver and releases it into the small intestine when fats are present there. Bile is an emulsifier that breaks up fat into micelles and allows more efficient digestion by lipases. If the gallbladder is removed, fat digestion will be more difficult, but digestion of other molecules would not be affected.

211. C. Mitosis is the normal process by which cells divide to replace old, worn-out cells or simply to allow growth of an organism. Fission is cell division in prokaryotes (choice A is wrong), and meiosis only occurs in germ-line cells (sperm and ova, choice B is wrong). Cell growth is limited by surface-to-volume ratios. Cells could not grow bigger to replace damaged tissue (choice D is wrong).

212. A. Compound I is an amine and can hydrogen bond with water molecules, making it fairly soluble. Compound II is completely nonpolar and will not dissolve in water, a polar solvent.

213. C. Cholesterol is a lipid that is the precursor for all steroid hormones, such as testosterone. Insulin is a protein (choice A is wrong), glycogen is the storage form of glucose (choice B is wrong), and DNA is a nucleic acid (choice D is wrong).

214. C. 2-Deoxythymidine can only be incorporated into DNA and not RNA. Thus if an increase is seen in the incorporated amount of this radiolabeled substance (as is shown in the graph), DNA synthesis must be occurring (choice C is correct and D is wrong). Certainly one can assume that either mitosis or meiosis (depending on whether these were somatic or germ line cells) is occurring as well, since the cells are described as “actively dividing,” but the incorporation of thymidine clearly indicates DNA synthesis, and choice C is a better answer than either choice A or B.